Site Loader

Содержание

Общее сопротивление цепи — правила, формулы и примеры вычисления

Величина, благодаря которой проводник способен не пропускать через себя ток или ограничивать его прохождение, называется электрическим сопротивлением цепи. Общее её значение для замкнутой схемы определяется с помощью формул. Их вид зависит от типа соединения элементов. Кроме того, если известны определённые характеристики, найти параметр можно, используя закон Ома для участка цепи.

Общие сведения

Прохождение электрического тока через проводник зависит от его проводимости. Это параметр пропорционален силе тока. Другими словами, он определяет способность вещества пропускать через себя электричество без потерь. Зависит проводимость от физических свойств материала, температуры, степени воздействия внешних сил. Обратной ей величиной является сопротивление, то есть характеристика проводника, показывающая его возможность сопротивляться прохождению тока.

Связь между фундаментальными параметрами электротока экспериментально установил Симон Ом. Он выяснил, что сила тока в замкнутой цепи пропорциональна разности потенциалов (напряжению) и обратно пропорциональна сопротивлению: I = U / R. Так, если R равно нулю, то сила тока будет бесконечной.

Способность веществ препятствовать прохождению электротока используется при построении электрических цепей. Так, радиоэлемент, который называется резистором, установленный в определённом месте электроцепи, позволяет получить на нагрузке нужное значение напряжения или тока. Радиодеталь представляет собой двухполюсник, который имеет установленное значение сопротивления или может изменять его.

Реальная замкнутая электрическая цепь состоит из множества активных и пассивных радиоэлементов. Каждый из них обладает каким-то значением сопротивления. В этом случае говорят о внутреннем сопротивлении прибора.

Расчёт выходных характеристик цепи, а именно величин тока и напряжения, требует знания общего сопротивления всей замкнутой цепочки. Иными словами, все элементы, начиная от источника питания и заканчивая нагрузкой, заменяются эквивалентными резисторами. Для цепи сначала считают общее значение сопротивления, а затем вычисляют нужные характеристики. Относительно источника тока, нагрузки и других элементов каждый резистор может быть подключён:

  • последовательно;
  • параллельно.

Вид подключения влияет на общее сопротивление. Формула для его нахождения может быть довольно громоздкой из-за смешанного соединения, поэтому чаще расчёт ведётся в несколько этапов, на каждом из которых выполняется объединение одного или нескольких элементов.

Последовательное подключение

Для удобства при изображении разветвлённой электрической цепи все сопротивления чертят в виде прямоугольников, которые являются резисторами. У любого такого элемента можно выделить два вывода. Один является началом, а другой — концом. С учетом сказанного можно сформулировать определение для последовательного соединения проводников: подключение, при котором конец предыдущего элемента соединён с началом последующего, называют последовательным.

Любой проводник обладает электрическим сопротивлением. Целью преобразования является замена чередующейся последовательности одним резистором. При этом по своим электрическим свойствам он должен не отличаться от всей цепочки. Простыми словами это можно пояснить так: если взять два чёрных ящика, у которых есть по паре выводов, причём один будет содержать всю электроцепь, а другой быть её эквивалентом, то определить, в каком из них находится схема, а где эквивалент, будет невозможно.

При последовательном соединении происходят следующие явления. Пусть имеется прямая цепочка, содержащая n резисторов: R1 + R2 + … +Rn. Сила тока — это величина, которая равняется заряду, протекающему за единицу времени. Можно представить, что в первом резисторе значение электротока будет больше, чем во втором. В результате возникнет «пробка», и скорость движения зарядов замедлится.

В точке соединения элементов произойдёт накопление электронов, что приведёт в ней к росту напряжения. Соответственно, сила тока на первом резисторе будет уменьшаться, а на втором, наоборот, увеличиваться. Это приведёт к выравниванию количества проходящих через резисторы зарядов, поэтому сила тока практически за мгновение во всей последовательной цепи станет одинаковой.

Напряжение — это работа, выполняемая по переносу заряда. По закону сохранения энергии общее её значение равняется их сумме на различных этапах. Общую разность потенциалов можно будет определить, сложив напряжения на каждом элементе. Такой вид подключения описывается следующими выражениями:

  • I = I 1 = I 2 = … = In;
  • U = U1 + U2 + … +Un.

Эти равенства являются фундаментальными для нахождения параметров при повторении резисторов в цепи. Используя закон Ома, можно найти, чему будет равняться сопротивление цепи. Формула для его нахождения будет выглядеть так: Rпос = R 1 + R 2 +… + Rn.

Параллельное соединение

По распространённости такой вид соединения чаще встречается, чем последовательное подключение. При нём проводники соединены так, что начала всех резисторов сводятся в одну точку электрической цепи, а концы — в другую. Для того чтобы заменить разветвлённое подключение одним эквивалентным элементом, нужно знать, как правильно рассчитать ток и напряжение.

Пусть имеется цепь, состоящая из R1 + R2 + … +Rn параллельно включённых радиоэлементов. На неё подаётся напряжение U. На вход схемы поступает ток с силой I. Используя закон сохранения зарядов, можно выполнить следующие рассуждения: ток втекает в узел, к которому подсоединены начала всех резисторов, затем он растекается по их выводам.

В результате через первую ветвь потечёт ток I1, вторую — I2, в энную — In. Поскольку заряд не может пропасть, то какое его количество втекло в узел, такое же должно разойтись по всем ветвям для одного и того же момента времени. Значит, сумма токов на всех выводах будет равняться поступающему на них значению.

Электростатическое поле является потенциальным, то есть работа по перемещению заряда из одной точки в другую не зависит от траектории, по которой перемещается носитель. Следовательно, при переносе одного кулона по любой ветви нужно будет совершить одинаковую работу. Из приведённых рассуждений следует, что при параллельном соединении формулы, с помощью которых можно рассчитать характеристики электрической цепи, будут следующими:

  • I = I1 + I2 + … +In;
  • U1 = U2 = … = Un.

Таким образом, вычисление эквивалентного сопротивления, которым можно будет заменить всю цепь в соответствии с законом Ома, выполняется по формуле: 1 / R пар = 1 / R 1 + 1 / R 2 + … + 1 / Rn. Для одинаковых проводников при вычислении сопротивления можно использовать приведённую формулу. Это позволяет в некоторых случаях упростить расчёт.

Согласно правилу сложения дробей c одинаковым знаменателем можно записать равенство: 1 / R1 + 1 / R2 + … + 1 / Rn = N / R1. Отсюда следует, что Rпар = R1 / N, где N равно числу резисторов. По аналогии можно посчитать общее сопротивление по упрощённой формуле для двух элементов: (1 / R1) + (1 / R2) = (R 2 + R 1) / R 1 * R 2. Это довольно удобные формулы для практического применения.

Решение задач

Для вычисления сопротивления любого смешанного соединения нужно запомнить всего две формулы — выражения для нахождения величины при последовательном и параллельном подключении. Поочерёдно комбинируя их применение, сложную схему можно заменить одним сопротивлением. Но не всегда приходится применять формулы. Есть задания, в которых неизвестную величину можно вычислить в уме.

Например, пусть имеется параллельное подключение из четырёх резисторов. Сопротивления проводников равняются 10 Ом, 12 Ом, 15 Ом, 20 Ом. Нужно образовать из них резистор, не изменяющий характеристики электрической цепи. Чтобы выполнить расчёт в уме, следует каждый элемент представить в виде комбинации из 60-омных резисторов. Тогда к первому нужно будет добавить шесть, ко второму — пять, к третьему — четыре, к четвёртому — три. Общее количество резисторов получится 18. Значит, Rобщ = 60 / 18 = 10 / 3 = 3,3 Ом.

Из типовых задач, в которых необходимо найти сопротивление цепи, предлагающихся в школе на уроках физики, можно привести следующие:

  • Найдите ток в цепи, если вольтметр, подключённый к одному из трёх последовательно соединённых проводников, показывает 100 В. Сопротивление элементов составляет: R1 = R2 = 5 Ом, R3 = 15 Ом. В задаче три резистора подключены в линию, значит, их полное сопротивление равно: R = R 1 + R 2 + R 3 = 25 Ом. У вольтметра r внутреннее равно бесконечности. Следовательно, I = U / R = 100 / 25 = 4 A.
  • Каково будет сопротивление каждого из резисторов, если при их последовательном соединении ток равен 3A, а при параллельном — 16A. Напряжение в сети составляет 120 В. При первом способе соединения Iпосл = U / (R1 + R2), при втором Iпар = U / Rпар = U * (R1 + R2) / R1 * R2. Из первой формулы следует, что R1 + R2 = U / Iпос. Тогда: I пар = U 2 / Iпос * R 1 * R 2 → R 1 * R 2 = U 2 / I пар * I пос. Используя теорему Виета, можно составить квадратное уравнение. После его решения искомые величины будут равны: R 1 = 30 Ом, R 2 = 10 Ом.

  • Следует отметить, что приборы для измерения тока, напряжения и даже ёмкости используют особенности вычисления сопротивления цепи. Так, вольтметр имеет бесконечно большой внутренний импеданс, что позволяет подключать его параллельно к измеряемым точкам без внесения изменения в протекающий сигнал.

    Амперметр же, наоборот, характеризуется пренебрежимо малой величиной внутреннего сопротивления, поэтому и подключают его в разрыв линии, на которой выполняют измерения.

    Предыдущая

    ФизикаУдельное электрическое сопротивление — формула, обозначение и физический смысл

    Следующая

    ФизикаДоклад на тему: «Траектория, путь и перемещение» — пример реферата

    Как найти rобщ в цепи

    Последовательное соединение резисторов

    Последовательное соединениеэто соединение двух или более резисторов в форме цепи, в которой каждый отдельный резистор соединяется с другим отдельным резистором только в одной точке.

    Общее сопротивление R

    общ

    При таком соединении, через все резисторы проходит один и тот же электрический ток. Чем больше элементов на данном участке электрической цепи, тем «труднее» току протекать через него. Следовательно, при последовательном соединении резисторов их общее сопротивление увеличивается, и оно равно сумме всех сопротивлений.

    Напряжение при последовательном соединении

    Напряжение при последовательном соединении распределяется на каждый резистор согласно закону Ома:

    Т.е чем большее сопротивление резистора, тем большее напряжение на него падает.

    Параллельное соединение резисторов

    Параллельное соединениеэто соединение, при котором резисторы соединяются между собой обоими контактами. В результате к одной точке (электрическому узлу) может быть присоединено несколько резисторов.

    Общее сопротивление R

    общ

    При таком соединении, через каждый резистор потечет отдельный ток. Сила данного тока будет обратно пропорциональна сопротивлению резистора. В результате общая проводимость такого участка электрической цепи увеличивается, а общее сопротивление в свою очередь уменьшается.

    Таким образом, при параллельном подсоединении резисторов с разным сопротивлением, общее сопротивление будет всегда меньше значения самого маленького отдельного резистора.

    Формула общей проводимости при параллельном соединении резисторов:

    Формула эквивалентного общего сопротивления при параллельном соединении резисторов:

    Для двух одинаковых резисторов общее сопротивление будет равно половине одного отдельного резистора:

    Соответственно, для n одинаковых резисторов общее сопротивление будет равно значению одного резистора, разделенного на n.

    Напряжение при параллельном соединении

    Напряжение между точками A и B является как общим напряжением для всего участка цепи, так и напряжением, падающим на каждый резистор в отдельности. Поэтому при параллельном соединении на все резисторы упадет одинаковое напряжение.

    Электрический ток при параллельном соединении

    Через каждый резистор течет ток, сила которого обратно пропорциональна сопротивлению резистора. Для того чтобы узнать какой ток течет через определенный резистор, можно воспользоваться законом Ома:

    Смешанное соединение резисторов

    Смешанным соединением называют участок цепи, где часть резисторов соединяются между собой последовательно, а часть параллельно. В свою очередь, смешанное соединение бывает последовательного и параллельного типов.

    Общее сопротивление R

    общ

    Для того чтобы посчитать общее сопротивление смешанного соединения:

    • Цепь разбивают на участки с только пареллельным или только последовательным соединением.
    • Вычисляют общее сопротивление для каждого отдельного участка.
    • Вычисляют общее сопротивление для всей цепи смешанного соединения.

    Так это будет выглядеть для схемы 1:

    Также существует более быстрый способ расчета общего сопротивления для смешанного соединения. Можно, в соответствии схеме, сразу записывать формулу следующим образом:

    • Если резисторы соединяются последоватеьно — складывать.
    • Если резисторы соединяются параллельно — использовать условное обозначение «||».
    • Подставлять формулу для параллельного соединения где стоит символ «||».

    Так это будет выглядеть для схемы 1:

    После подстановки формулы параллельного соединения вместо «||»:

    Последовательное соединение резисторов

    Последовательное соединениеэто соединение двух или более резисторов в форме цепи, в которой каждый отдельный резистор соединяется с другим отдельным резистором только в одной точке.

    Общее сопротивление R

    общ

    При таком соединении, через все резисторы проходит один и тот же электрический ток. Чем больше элементов на данном участке электрической цепи, тем «труднее» току протекать через него. Следовательно, при последовательном соединении резисторов их общее сопротивление увеличивается, и оно равно сумме всех сопротивлений.

    Напряжение при последовательном соединении

    Напряжение при последовательном соединении распределяется на каждый резистор согласно закону Ома:

    Т.е чем большее сопротивление резистора, тем большее напряжение на него падает.

    Параллельное соединение резисторов

    Параллельное соединениеэто соединение, при котором резисторы соединяются между собой обоими контактами. В результате к одной точке (электрическому узлу) может быть присоединено несколько резисторов.

    Общее сопротивление R

    общ

    При таком соединении, через каждый резистор потечет отдельный ток. Сила данного тока будет обратно пропорциональна сопротивлению резистора. В результате общая проводимость такого участка электрической цепи увеличивается, а общее сопротивление в свою очередь уменьшается.

    Таким образом, при параллельном подсоединении резисторов с разным сопротивлением, общее сопротивление будет всегда меньше значения самого маленького отдельного резистора.

    Формула общей проводимости при параллельном соединении резисторов:

    Формула эквивалентного общего сопротивления при параллельном соединении резисторов:

    Для двух одинаковых резисторов общее сопротивление будет равно половине одного отдельного резистора:

    Соответственно, для n одинаковых резисторов общее сопротивление будет равно значению одного резистора, разделенного на n.

    Напряжение при параллельном соединении

    Напряжение между точками A и B является как общим напряжением для всего участка цепи, так и напряжением, падающим на каждый резистор в отдельности. Поэтому при параллельном соединении на все резисторы упадет одинаковое напряжение.

    Электрический ток при параллельном соединении

    Через каждый резистор течет ток, сила которого обратно пропорциональна сопротивлению резистора. Для того чтобы узнать какой ток течет через определенный резистор, можно воспользоваться законом Ома:

    Смешанное соединение резисторов

    Смешанным соединением называют участок цепи, где часть резисторов соединяются между собой последовательно, а часть параллельно. В свою очередь, смешанное соединение бывает последовательного и параллельного типов.

    Общее сопротивление R

    общ

    Для того чтобы посчитать общее сопротивление смешанного соединения:

    • Цепь разбивают на участки с только пареллельным или только последовательным соединением.
    • Вычисляют общее сопротивление для каждого отдельного участка.
    • Вычисляют общее сопротивление для всей цепи смешанного соединения.

    Так это будет выглядеть для схемы 1:

    Также существует более быстрый способ расчета общего сопротивления для смешанного соединения. Можно, в соответствии схеме, сразу записывать формулу следующим образом:

    • Если резисторы соединяются последоватеьно — складывать.
    • Если резисторы соединяются параллельно — использовать условное обозначение «||».
    • Подставлять формулу для параллельного соединения где стоит символ «||».

    Так это будет выглядеть для схемы 1:

    После подстановки формулы параллельного соединения вместо «||»:

    Как я и обещал в статье про переменные резисторы (ссылка), сегодня речь пойдет о возможных способах соединения резисторов, в частности о последовательном соединении и о параллельном.

    Последовательное соединение резисторов.

    Давайте начнем с рассмотрения цепей, элементы которой соединены последовательно. И хоть мы и будем рассматривать только резисторы в качестве элементов цепи в данной статье, но правила, касающиеся напряжений и токов при разных соединениях будут справедливы и для других элементов. Итак, первая цепь, которую мы будем разбирать выглядит следующим образом:

    Здесь у нас классический случай последовательного соединения – два последовательно включенных резистора. Но не будем забегать вперед и рассчитывать общее сопротивление цепи, а для начала рассмотрим все напряжения и токи. Итак, первое правило заключается в том, что протекающие по всем проводникам токи при последовательном соединении равны между собой:

    А для определения общего напряжения при последовательном соединении, напряжения на отдельных элементах необходимо просуммировать:

    В то же время, по закону Ома для напряжений, сопротивлений и токов в данной цепи справедливы следующие соотношения:

    Тогда для вычисления общего напряжения можно будет использовать следующее выражение:

    Но для общего напряжение также справедлив закон Ома:

    Здесь – это общее сопротивление цепи, которое исходя из двух формул для общего напряжения равно:

    Таким образом, при последовательном соединении резисторов общее сопротивление цепи будет равно сумме сопротивлений всех проводников.

    Например для следующей цепи:

    Общее сопротивление будет равно:

    Количество элементов значения не имеет, правило, по которому мы определяем общее сопротивление будем работать в любом случае 🙂 А если при последовательном соединении все сопротивления равны (), то общее сопротивление цепи составит:

    в данной формуле равно количеству элементов цепи.

    С последовательным соединением резисторов мы разобрались, давайте перейдем к параллельному.

    Параллельное соединение резисторов.

    При параллельном соединении напряжения на проводниках равны:

    А для токов справедливо следующее выражение:

    То есть общий ток разветвляется на две составляющие, а его значение равно сумме всех составляющих. По закону Ома:

    Подставим эти выражения в формулу общего тока:

    А по закону Ома ток:

    Приравниваем эти выражения и получаем формулу для общего сопротивления цепи:

    Данную формулу можно записать и несколько иначе:

    Таким образом, при параллельном соединении проводников величина, обратная общему сопротивлению цепи, равна сумме величин, обратных сопротивлениям параллельно включенных проводников.

    Аналогичная ситуация будет наблюдаться и при большем количестве проводников, соединенных параллельно:

    Смешанное соединение резисторов.

    Помимо параллельного и последовательного соединений резисторов существует еще смешанное соединение. Из названия уже понятно, что при таком соединении в цепи присутствуют резисторы, соединенные как параллельно, так и последовательно. Вот пример такой цепи:

    Давайте рассчитаем общее сопротивление цепи. Начнем с резисторов и – они соединены параллельно. Мы можем рассчитать общее сопротивление для этих резисторов и заменить их в схеме одним единственным резистором :

    Теперь у нас образовались две группы последовательно соединенных резисторов:

    Заменим эти две группы двумя резисторами, сопротивление которых равно:

    Как видите, схема стала уже совсем простой ) Заменим группу параллельно соединенных резисторов и одним резистором :

    И в итоге у нас на схеме осталось только два резистора соединенных последовательно:

    Общее сопротивление цепи получилось равным:

    Таким вот образом достаточно большая схема свелась к простейшему последовательному соединению двух резисторов 😉

    Тут стоит отметить, что некоторые схемы невозможно так просто преобразовать и определить общее сопротивление – для таких схем нужно использовать правила Кирхгофа, о которых мы обязательно поговорим в будущих статьях. А сегодняшняя статья на этом подошла к концу, до скорых встреч на нашем сайте!

    Формула общего сопротивления при последовательном соединении. Электрическая цепь с последовательным соединением элементов

    Отдельные проводники электрической цепи могут быть соединены между собой последовательно, параллельно и смешанно. При этом последовательное и параллельное соединение проводников являются основными видами соединений, а смешанное соединение это их совокупность.

    Последовательным соединением проводников называется такое соединение, когда конец первого проводника соединен с началом второго, конец второго проводника соединен с началом третьего и так далее (рисунок 1).

    Рисунок 1. Схема последовательного соединения проводников

    Общее сопротивление цепи, состоящее из нескольких последовательно соединенных проводников, равно сумме сопротивлений отдельных проводников:

    r = r 1 + r 2 + r 3 + … + r n .

    Ток на отдельных участках последовательной цепи везде одинаков:

    I 1 = I 2 = I 3 = I .

    Видео 1. Последовательное соединение проводников

    Пример 1. На рисунке 2 представлена электрическая цепь, состоящая из трех последовательно включенных сопротивлений r 1 = 2 Ом, r 2 = 3 Ом, r 3 = 5 Ом. Требуется определить показания вольтметров V 1 , V 2 , V 3 и V 4 , если ток в цепи равен 4 А.

    Сопротивление всей цепи

    r = r 1 + r 2 + r 3 = 2 + 3 + 5 =10 Ом.

    Рисунок 2. Схема измерения напряжений на отдельных участках электрической цепи

    В сопротивлении r 1 при протекании тока будет падение напряжения:

    U 1 = I × r 1 = 4 × 2 = 8 В.

    Вольтметр V 1 , включенный между точками а и б , покажет 8 В.

    В сопротивлении r 2 также происходит падение напряжения:

    U 2 = I × r 2 = 4 × 3 = 12 В.

    Вольтметр V 2 , включенный между точками в и г , покажет 12 В.

    Падение напряжения в сопротивлении r 3:

    U 3 = I × r 3 = 4 × 5 = 20 В.

    Вольтметр V 3 , включенный между точками д и е , покажет 20 В.

    Если вольтметр присоединить одним концом к точке а , другим концом к точке г , то он покажет разность потенциалов между этими точками, равную сумме падений напряжения в сопротивлениях r 1 и r 2 (8 + 12 = 20 В).

    Таким образом, вольтметр V , измеряющий напряжение на зажимах цепи и включенный между точками а и е , покажет разность потенциалов между этими точками или сумму падений напряжения в сопротивлениях r 1 , r 2 и r 3 .

    Отсюда видно, что сумма падений напряжения на отдельных участках электрической цепи равна напряжению на зажимах цепи.

    Так как при последовательном соединении ток цепи на всех участках одинаков, то падение напряжения пропорционально сопротивлению данного участка.

    Пример 2. Три сопротивления 10, 15 и 20 Ом соединены последовательно, как показано на рисунке 3. Ток в цепи 5 А. Определить падение напряжения на каждом сопротивлении.

    U 1 = I × r 1 = 5 ×10 = 50 В,
    U 2 = I × r 2 = 5 ×15 = 75 В,
    U 3 = I × r 3 = 5 ×20 = 100 В.

    Рисунок 3. К примеру 2

    Общее напряжение цепи равно сумме падений напряжений на отдельных участках цепи:

    U = U 1 + U 2 + U 3 = 50 + 75 + 100 = 225 В.

    Параллельное соединение проводников

    Параллельным соединением проводников называется такое соединение, когда начала всех проводников соединены в одну точку, а концы проводников – в другую точку (рисунок 4). Начало цепи присоединяется к одному полюсу источника напряжения, а конец цепи – к другому полюсу.

    Из рисунка видно, что при параллельном соединении проводников для прохождения тока имеется несколько путей. Ток, протекая к точке разветвления А , растекается далее по трем сопротивлениям и равен сумме токов, уходящих от этой точки:

    I = I 1 + I 2 + I 3 .

    Если токи, приходящие к точке разветвления, считать положительными, а уходящие – отрицательными, то для точки разветвления можно написать:

    то есть алгебраическая сумма токов для любой узловой точки цепи всегда равна нулю. Это соотношение, связывающее токи в любой точке разветвления цепи, называется первым законом Кирхгофа . Определение первого закона Кирхгофа может звучать и в другой формулировке, а именно: сумма токов втекающих в узел электрической цепи равна сумме токов вытекающих из этого узла.

    Видео 2. Первый закон Кирхгофа

    Обычно при расчете электрических цепей направление токов в ветвях, присоединенных к какой либо точке разветвления, неизвестны. Поэтому для возможности самой записи уравнения первого закона Кирхгофа нужно перед началом расчета цепи произвольно выбрать так называемые положительные направления токов во всех ее ветвях и обозначить их стрелками на схеме.

    Пользуясь законом Ома, можно вывести формулу для подсчета общего сопротивления при параллельном соединении потребителей.

    Общий ток, приходящий к точке А , равен:

    Токи в каждой из ветвей имеют значения:

    По формуле первого закона Кирхгофа

    I = I 1 + I 2 + I 3

    Вынося U в правой части равенства за скобки, получим:

    Сокращая обе части равенства на U , получим формулу подсчета общей проводимости:

    g = g 1 + g 2 + g 3 .

    Таким образом, при параллельном соединении увеличивается не сопротивление, а проводимость .

    Пример 3. Определить общее сопротивление трех параллельно включенных сопротивлений, если r 1 = 2 Ом, r 2 = 3 Ом, r 3 = 4 Ом.

    Пример 4. Пять сопротивлений 20, 30 ,15, 40 и 60 Ом включены параллельно в сеть. Определить общее сопротивление:

    Следует заметить, что при подсчете общего сопротивления разветвления оно получается всегда меньше, чем самое меньшее сопротивление, входящее в разветвление.

    Если сопротивления, включенные параллельно, равны между собой, то общее сопротивление r цепи равно сопротивлению одной ветви r 1 , деленному на число ветвей n :

    Пример 5. Определить общее сопротивление четырех параллельно включенных сопротивлений по 20 Ом каждое:

    Для проверки попробуем найти сопротивление разветвления по формуле:

    Как видим, ответ получается тот же.

    Пример 6. Пусть требуется определить токи в каждой ветви при параллельном их соединении, изображенном на рисунке 5, а .

    Найдем общее сопротивление цепи:

    Теперь все разветвления мы можем изобразить упрощенно как одно сопротивление (рисунок 5, б ).

    Падение напряжения на участке между точками А и Б будет:

    U = I × r = 22 × 1,09 = 24 В.

    Возвращаясь снова к рисунку 5, а видим, что все три сопротивления окажутся под напряжением 24 В, так как они включены между точками А и Б .

    Рассматривая первую ветвь разветвления с сопротивлением r 1 , мы видим, что напряжение на этом участке 24 В, сопротивление участка 2 Ом. По закону Ома для участка цепи ток на этом участке будет:

    Ток второй ветви

    Ток третьей ветви

    Проверим по первому закону Кирхгофа

    Последовательное, параллельное и смешанное соединения резисторов. Значительное число приемников, включенных в электрическую цепь (электрические лампы, электронагревательные приборы и др.), можно рассматривать как некоторые элементы, имеющие определенное сопротивление. Это обстоятельство дает нам возможность при составлении и изучении электрических схем заменять конкретные приемники резисторами с определенными сопротивлениями. Различают следующие способы соединения резисторов (приемников электрической энергии): последовательное, параллельное и смешанное.

    Последовательное соединение резисторов . При последовательном соединении нескольких резисторов конец первого резистора соединяют с началом второго, конец второго — с началом третьего и т. д. При таком соединении по всем элементам последовательной цепи проходит
    один и тот же ток I.
    Последовательное соединение приемников поясняет рис. 25, а.
    .Заменяя лампы резисторами с сопротивлениями R1, R2 и R3, получим схему, показанную на рис. 25, б.
    Если принять, что в источнике Ro = 0, то для трех последовательно соединенных резисторов согласно второму закону Кирхгофа можно написать:

    E = IR 1 + IR 2 + IR 3 = I(R 1 + R 2 + R 3) = IR эк (19)

    где R эк = R 1 + R 2 + R 3 .
    Следовательно, эквивалентное сопротивление последовательной цепи равно сумме сопротивлений всех последовательно соединенных резисторов.Так как напряжения на отдельных участках цепи согласно закону Ома: U 1 =IR 1 ; U 2 = IR 2 , U 3 = IR з и в данном случае E = U, то длярассматриваемой цепи

    U = U 1 + U 2 +U 3 (20)

    Следовательно, напряжение U на зажимах источника равно сумме напряжений на каждом из последовательно включенных резисторов.
    Из указанных формул следует также, что напряжения распределяются между последовательно соединенными резисторами пропорционально их сопротивлениям:

    U 1: U 2: U 3 = R 1: R 2: R 3 (21)

    т. е. чем больше сопротивление какого-либо приемника в последовательной цепи, тем больше приложенное к нему напряжение.

    В случае если последовательно соединяются несколько, например п, резисторов с одинаковым сопротивлением R1, эквивалентное сопротивление цепи Rэк будет в п раз больше сопротивления R1, т. е. Rэк = nR1. Напряжение U1 на каждом резисторе в этом случае в п раз меньше общего напряжения U:

    При последовательном соединении приемников изменение сопротивления одного из них тотчас же влечет за собой изменение напряжения на других связанных с ним приемниках. При выключении или обрыве электрической цепи в одном из приемников и в остальных приемниках прекращается ток. Поэтому последовательное соединение приемников применяют редко — только в том случае, когда напряжение источника электрической энергии больше номинального напряжения, на которое рассчитан потребитель. Например, напряжение в электрической сети, от которой питаются вагоны метрополитена, составляет 825 В, номинальное же напряжение электрических ламп, применяемых в этих вагонах, 55 В. Поэтому в вагонах метрополитена электрические лампы включают последовательно по 15 ламп в каждой цепи.
    Параллельное соединение резисторов . При параллельном соединении нескольких приемников они включаются между двумя точками электрической цепи, образуя параллельные ветви (рис. 26, а). Заменяя

    лампы резисторами с сопротивлениями R1, R2, R3, получим схему, показанную на рис. 26, б.
    При параллельном соединении ко всем резисторам приложено одинаковое напряжение U. Поэтому согласно закону Ома:

    I 1 =U/R 1 ; I 2 =U/R 2 ; I 3 =U/R 3 .

    Ток в неразветвленной части цепи согласно первому закону Кирхгофа I = I 1 +I 2 +I 3 , или

    I = U / R 1 + U / R 2 + U / R 3 = U (1/R 1 + 1/R 2 + 1/R 3) = U / R эк (23)

    Следовательно, эквивалентное сопротивление рассматриваемой цепи при параллельном соединении трех резисторов определяется формулой

    1/R эк = 1/R 1 + 1/R 2 + 1/R 3 (24)

    Вводя в формулу (24) вместо значений 1/R эк, 1/R 1 , 1/R 2 и 1/R 3 соответствующие проводимости G эк, G 1 , G 2 и G 3 , получим: эквивалентная проводимость параллельной цепи равна сумме проводимостей параллельно соединенных резисторов :

    G эк = G 1 + G 2 +G 3 (25)

    Таким образом, при увеличении числа параллельно включаемых резисторов результирующая проводимость электрической цепи увеличивается, а результирующее сопротивление уменьшается.
    Из приведенных формул следует, что токи распределяются между параллельными ветвями обратно пропорционально их электрическим сопротивлениям или прямо пропорционально их проводимостям. Например, при трех ветвях

    I 1: I 2: I 3 = 1/R 1: 1/R 2: 1/R 3 = G 1 + G 2 + G 3 (26)

    В этом отношении имеет место полная аналогия между распределением токов по отдельным ветвям и распределением потоков воды по трубам.
    Приведенные формулы дают возможность определить эквивалентное сопротивление цепи для различных конкретных случаев. Например, при двух параллельно включенных резисторах результирующее сопротивление цепи

    R эк =R 1 R 2 /(R 1 +R 2)

    при трех параллельно включенных резисторах

    R эк =R 1 R 2 R 3 /(R 1 R 2 +R 2 R 3 +R 1 R 3)

    При параллельном соединении нескольких, например n, резисторов с одинаковым сопротивлением R1 результирующее сопротивление цепи Rэк будет в n раз меньше сопротивления R1, т.е.

    R эк = R1 / n (27)

    Проходящий по каждой ветви ток I1, в этом случае будет в п раз меньше общего тока:

    I1 = I / n (28)

    При параллельном соединении приемников, все они находятся под одним и тем же напряжением, и режим работы каждого из них не зависит от остальных. Это означает, что ток, проходящий по какому-либо из приемников, не будет оказывать существенного влияния на другие приемники. При всяком выключении или выходе из строя любого приемника остальные приемники остаются вклю-

    ченными. Поэтому параллельное соединение имеет существенные преимущества перед последовательным, вследствие чего оно получило наиболее широкое распространение. В частности, электрические лампы и двигатели, предназначенные для работы при определенном (номинальном) напряжении, всегда включают параллельно.
    На электровозах постоянного тока и некоторых тепловозах тяговые двигатели в процессе регулирования скорости движения нужно включать под различные напряжения, поэтому они в процессе разгона переключаются с последовательного соединения на параллельное.

    Смешанное соединение резисторов . Смешанным соединением называется такое соединение, при котором часть резисторов включается последовательно, а часть — параллельно. Например, в схеме рис. 27, а имеются два последовательно включенных резистора сопротивлениями R1 и R2, параллельно им включен резистор сопротивлением Rз, а резистор сопротивлением R4 включен последовательно с группой резисторов сопротивлениями R1, R2 и R3.
    Эквивалентное сопротивление цепи при смешанном соединении обычно определяют методом преобразования, при котором сложную цепь последовательными этапами преобразовывают в простейшую. Например, для схемы рис. 27, а вначале определяют эквивалентное сопротивление R12 последовательно включенных резисторов с сопротивлениями R1 и R2: R12 = R1 + R2. При этом схема рис. 27, а заменяется эквивалентной схемой рис. 27, б. Затем определяют эквивалентное сопротивление R123 параллельно включенных сопротивлений и R3 по формуле

    R 123 =R 12 R 3 /(R 12 +R 3)=(R 1 +R 2)R 3 /(R 1 +R 2 +R 3).

    При этом схема рис. 27, б заменяется эквивалентной схемой рис. 27, в. После этого находят эквивалентное сопротивление всей цепи суммированием сопротивления R123 и последовательно включенного с ним сопротивления R4:

    R эк = R 123 + R 4 = (R 1 + R 2) R 3 / (R 1 + R 2 + R 3) + R 4

    Последовательное, параллельное и смешанное соединения широко применяют для изменения сопротивления пусковых реостатов при пуске э. п. с. постоянного тока.

    Параллельные соединения резисторов, формула расчёта которых выводится из закона Ома и правил Кирхгофа, являются наиболее распространённым типом включения элементов в электрическую цепь. При параллельном соединении проводников два или несколько элементов объединяются своими контактами с обеих из сторон соответственно. Подключение их к общей схеме осуществляется именно этими узловыми точками.

    Gif?x15027″ alt=»Общий вид»>

    Общий вид

    Особенности включения

    Включённые таким образом проводники нередко входят в состав сложных цепочек, содержащих, помимо этого, последовательное соединение отдельных участков.

    Для такого включения типичны следующие особенности:

    • Общее напряжение в каждой из ветвей будет иметь одно и то же значение;
    • Протекающий в любом из сопротивлений электрический ток всегда обратно пропорционален величине их номинала.

    В частном случае, когда все включённые в параллель резисторы имеют одинаковые номинальные значения, протекающие по ним «индивидуальные» токи также будут равны между собой.

    Расчёт

    Сопротивления ряда соединённых в параллель проводящих элементов определяются по общеизвестной форме расчёта, предполагающей сложение их проводимостей (обратных сопротивлению величин).

    Протекающий в каждом из отдельных проводников ток в соответствие с законом Ома, может быть найден по формуле:

    I= U/R (одного из резисторов).

    После ознакомления с общими принципами обсчёта элементов сложных цепочек можно перейти к конкретным примерам решения задач данного класса.

    Типичные подключения

    Пример №1

    Нередко для решения стоящей перед конструктором задачи требуется путём объединения нескольких элементов получить в итоге конкретное сопротивление. При рассмотрении простейшего варианта такого решения допустим, что общее сопротивление цепочки из нескольких элементов должно составлять 8 Ом. Этот пример нуждается в отдельном рассмотрении по той простой причине, что в стандартном ряду сопротивлений номинал в 8 Ом отсутствует (есть только 7,5 и 8,2 Ом).

    Решение этой простейшей задачи удаётся получить за счёт соединения двух одинаковых элементов с сопротивлениями по 16 Ом каждое (такие номиналы в резистивном ряду существуют). Согласно приводимой выше формуле общее сопротивление цепочки в этом случае вычисляется очень просто.

    Из неё следует:

    16х16/32=8 (Ом), то есть как раз столько, сколько требовалось получить.

    Таким сравнительно простым способом удаётся решить задачу формирования общего сопротивления, равного 8-ми Омам.

    Пример №2

    В качестве ещё одного характерного примера образования требуемого сопротивления можно рассмотреть построение схемы, состоящей из 3-х резисторов.

    Общее значение R такого включения может быть рассчитано по формуле последовательного и параллельного соединения в проводниках.

    Gif?x15027″ alt=»Пример»>

    В соответствии с указанными на картинке значениями номиналов, общее сопротивление цепочки будет равно:

    1/R = 1/200+1/220+1/470 = 0,0117;

    R=1/0,0117 = 85,67Ом.

    В итоге находим суммарное сопротивление всей цепочки, получаемой при параллельном соединении трёх элементов с номинальными значениями 200, 240 и 470 Ом.

    Важно! Указанный метод применим и при расчёте произвольного числа соединенных в параллель проводников или потребителей.

    Также необходимо отметить, что при таком способе включения различных по величине элементов общее сопротивление будет меньше, чем у самого малого номинала.

    Расчёт комбинированных схем

    Рассмотренный метод может применяться и при расчёте сопротивления более сложных или комбинированных схем, состоящих из целого набора компонентов. Их иногда называют смешанными, поскольку при формировании цепочек используются сразу оба способа. Смешанное соединение резисторов представлено на размещенном ниже рисунке.

    Gif?x15027″ alt=»Смешанная схема»>

    Смешанная схема

    Для упрощения расчета сначала разбиваем все резисторы по типу включения на две самостоятельные группы. Одна из них представляет собой последовательное соединение, а вторая – имеет вид подключения параллельного типа.

    Из приведённой схемы видно, что элементы R2 и R3 соединяются последовательно (они объединены в группу 2), которая, в свою очередь, включена в параллель с резистором R1, принадлежащим группе 1.

    Последовательным называется такое соединение резисторов, когда конец одного проводника соединяется с началом другого и т.д. (рис. 1). При последовательном соединении сила тока на любом участке электрической цепи одинакова. Это объясняется тем, что заряды не могут накапливаться в узлах цепи. Их накопление привело бы к изменению напряженности электрического поля, а следовательно, и к изменению силы тока. Поэтому

    \(~I = I_1 = I_2 .\)

    Амперметр А измеряет силу тока в цепи и обладает малым внутренним сопротивлением (R A → 0).

    Включенные вольтметры V 1 и V 2 измеряют напряжение U 1 и U 2 на сопротивлениях R 1 и R 2 . Вольтметр V измеряет подведенное к клеммам Μ и N напряжение U .n R_i .\)

    Если сопротивления отдельных резисторов равны между собой, т.е. R 1 = R 2 = … = R n , то общее сопротивление этих резисторов при последовательном соединении в n раз больше сопротивления одного резистора: R = nR 1 .

    При последовательном соединении резисторов справедливо соотношение \(~\frac{U_1}{U_2} = \frac{R_1}{R_2}\), т.е. напряжения на резисторах прямо пропорциональны сопротивлениям.

    Параллельным называется такое соединение резисторов, когда одни концы всех резисторов соединены в один узел, другие концы — в другой узел (рис. 2). Узлом называется точка разветвленной цепи, в которой сходятся более двух проводников. При параллельном соединении резисторов к точкам Μ и N подключен вольтметр. Он показывает, что напряжения на отдельных участках цепи с сопротивлениями R 1 и R 2 равны. Это объясняется тем, что работа сил стационарного электрического поля не зависит от формы траектории:

    \(~U = U_1 = U_2 .n \frac{1}{R_i} .\)

    Если сопротивления всех n параллельно соединенных резисторов одинаковы и равны R 1 то \(~\frac 1R = \frac{n}{R_1}\) . Откуда \(~R = \frac{R_1}{n}\) .

    Сопротивление цепи, состоящей из n одинаковых параллельно соединенных резисторов, в n раз меньше сопротивления каждого из них.

    При параллельном соединении резисторов справедливо соотношение \(~\frac{I_1}{I_2} = \frac{R_2}{R_1}\), т.е. силы токов в ветвях параллельно соединенной цепи обратно пропорциональны сопротивлениям ветвей.

    Литература

    Аксенович Л. А. Физика в средней школе: Теория. Задания. Тесты: Учеб. пособие для учреждений, обеспечивающих получение общ. сред, образования / Л. А. Аксенович, Н.Н.Ракина, К. С. Фарино; Под ред. К. С. Фарино. — Мн.: Адукацыя i выхаванне, 2004. — C. 257-259.

    Проверим справедливость показанных здесь формул на простом эксперименте.

    Возьмём два резистора МЛТ-2 на 3 и 47 Ом и соединим их последовательно. Затем измерим общее сопротивление получившейся цепи цифровым мультиметром. Как видим оно равно сумме сопротивлений резисторов, входящих в эту цепочку.


    Замер общего сопротивления при последовательном соединении

    Теперь соединим наши резисторы параллельно и замерим их общее сопротивление.


    Измерение сопротивления при параллельном соединении

    Как видим, результирующее сопротивление (2,9 Ом) меньше самого меньшего (3 Ом), входящего в цепочку. Отсюда вытекает ещё одно известное правило, которое можно применять на практике:

    При параллельном соединении резисторов общее сопротивление цепи будет меньше наименьшего сопротивления, входящего в эту цепь.

    Что ещё нужно учитывать при соединении резисторов?

    Во-первых, обязательно учитывается их номинальная мощность. Например, нам нужно подобрать замену резистору на 100 Ом и мощностью 1 Вт . Возьмём два резистора по 50 Ом каждый и соединим их последовательно. На какую мощность рассеяния должны быть рассчитаны эти два резистора?

    Поскольку через последовательно соединённые резисторы течёт один и тот же постоянный ток (допустим 0,1 А ), а сопротивление каждого из них равно 50 Ом , тогда мощность рассеивания каждого из них должна быть не менее 0,5 Вт . В результате на каждом из них выделится по 0,5 Вт мощности. В сумме это и будет тот самый 1 Вт .

    Данный пример достаточно грубоват. Поэтому, если есть сомнения, стоит брать резисторы с запасом по мощности.

    Подробнее о мощности рассеивания резистора читайте .

    Во-вторых, при соединении стоит использовать однотипные резисторы, например, серии МЛТ. Конечно, нет ничего плохого в том, чтобы брать разные. Это лишь рекомендация.

    Калькулятор соединения резисторов онлайн. Параллельное соединение резисторов

    В каждой электрической схеме присутствует резистор, имеющий сопротивление электрическому току. Резисторы бывают двух типов: постоянные и переменные. Во время разработки любой электрической схемы и ремонта электронных изделий часто приходится применять резистор, обладающий необходимым номиналом.

    Несмотря на то что для резисторов предусмотрены различные номиналы , может случиться так, что не будет возможности найти необходимый или же вообще ни один элемент не сможет обеспечить требуемый показатель.

    Решением этой проблемы может стать применение последовательного и параллельного соединения. Ознакомившись с этой статьей, вы узнаете об особенностях выполнения расчета и подбора различных номиналов сопротивлений.

    Параллельное соединение: общая информация

    Часто при изготовлении какого-либо устройства используют резисторы, которые соединяются в соответствии с последовательной схемой. Эффект от применения такого варианта сборки сводится к увеличению общего сопротивления цепи. Для данного варианта соединения элементов создаваемое ими сопротивление рассчитывается как сумма номиналов. Если же сборка деталей выполняется по параллельной схеме, то здесь потребуется рассчитать сопротивление , используя нижеописанные формулы.

    К схеме параллельного соединения прибегают в ситуации, когда стоит задача по снижению суммарного сопротивления, а, помимо этого, увеличения мощности для группы элементов, подключенных по параллельной схеме, которое должно быть больше, чем при их отдельном подключении.

    Расчет сопротивления

    В случае подключения деталей друг с другом, с применением параллельной схемы для расчета суммарного сопротивления, будет использоваться следующая формула:

    R(общ)=1/(1/R1+1/R2+1/R3+1/Rn).

    • R1- R3 и Rn – резисторы, подсоединенные по параллельной схеме.

    Причем, если цепь создается на основе только двух элементов, то для определения суммарного номинального сопротивления следует использовать такую формулу:

    R(общ)=R1*R2/R1+R2.

    • R(общ) – суммарное сопротивление;
    • R1 и R2 – резисторы, подсоединенные по параллельной схеме.

    Видео: Пример расчёта сопротивления

    Универсальная схема расчета

    Применительно к радиотехнике следует уделить внимание одному важному правилу: если подключаемые друг к другу элементы по параллельной схеме имеют одинаковый показатель , то для расчета суммарного номинала необходимо общее значение разделить на число подключенных узлов:

    • R(общ) – суммарное значение сопротивления;
    • R – номинал резистора, подсоединенного по параллельной схеме;
    • n – число подключенных узлов.

    Особое внимание следует обратить на то, что конечный показатель сопротивления в случае использования параллельной схемы подключения обязательно будет меньше по сравнению с номиналом любого элемента, подключаемого в цепь.

    Пример расчёта

    Для большей наглядности можно рассмотреть следующий пример: допустим, у нас есть три резистора, чьи номиналы соответственно равны 100, 150 и 30 Ом. Если воспользоваться первой формулой для определения общего номинала, то получим следующее:

    R(общ)=1/(1/100+1/150+1/30)=

    1/(0,01+0,007+0,03)=1/0,047=21,28Ом.

    Если выполнить несложные расчеты, то можно получить следующее: для цепи, включающей в себя три детали, где наименьший показатель сопротивления составляет 30 Ом, результирующее значение номинала будет равно 21,28 Ом. Этот показатель будет меньше минимального значения номинала в цепи практически на 30%.

    Важные нюансы

    Обычно для резисторов параллельное соединение применяется тогда, когда стоит задача по созданию сопротивления большей мощности. Для ее решения потребуются резисторы, которые должны иметь равные показатели сопротивления и мощности. При таком варианте определить общую мощность можно следующим образом : мощность одного элемента необходимо перемножить с суммарным числом всех резисторов, из которых состоит цепь, подсоединенных друг с другом в соответствии с параллельной схемой.

    Скажем, если нами будут использоваться пять резисторов, чей номинал составляет 100 Ом, а мощность каждого равна 1 Вт, которые присоединены друг к другу в соответствии с параллельной схемой, то суммарный показатель сопротивления будет равен 20 Ом, а мощность составит 5 Вт.

    Если взять те же резисторы, но подсоединить их в соответствии с последовательной схемой, то конечная мощность составит 5 Вт, а суммарный номинал будет равен 500 Ом.

    Видео: Правильное подключение светодиодов

    Параллельная схема подключения резисторов очень востребована по той причине, что часто возникает задача по созданию такого номинала, которого невозможно добиться при помощи простого параллельного соединения. При этом процедура расчета этого параметра отличается достаточной сложностью , где необходимо учитывать разные параметры.

    Здесь важная роль отводится не только количеству подключаемых элементов, но и рабочим параметрам резисторов — прежде всего, сопротивлению и мощности. Если один из подключаемых элементов будет иметь неподходящий показатель, то это не позволит эффективно решить задачу по созданию требуемого номинала в цепи.

    Параллельное соединение резисторов — одно из двух видов электрических соединений, когда оба вывода одного резистора соединены с соответствующими выводами другого резистора или резисторов. Зачастую или параллельно для того, чтобы создать более сложные электронные схемы.

    Схема параллельного соединения показан на рисунке ниже. При параллельном соединении резисторов, напряжение на всех резисторах будет одинаковым, а протекающий через них ток будет пропорционален их сопротивлению:

    Формула параллельного соединения резисторов

    Общее сопротивление нескольких резисторов соединенных параллельно определяется по следующей формуле:

    Ток, протекающий через отдельно взятый резистор, согласно , можно найти по формуле:

    Параллельное соединение резисторов — расчет

    Пример №1

    При разработке устройства, возникла необходимость установить резистор с сопротивлением 8 Ом. Если мы просмотрим весь номинальный ряд стандартных значений резисторов, то мы увидим, что резистора с сопротивлением в 8 Ом в нем нет.

    Выходом из данной ситуации будет использование двух параллельно соединенных резисторов. Эквивалентное значение сопротивления для двух резисторов соединенных параллельно рассчитывается следующим образом:

    Данное уравнение показывает, что если R1 равен R2, то сопротивление R составляет половину сопротивления одного из двух резисторов. При R = 8 Ом, R1 и R2 должны, следовательно, иметь значение 2 × 8 = 16 Ом.
    Теперь проведем проверку, рассчитав общее сопротивление двух резисторов:

    Таким образом, мы получили необходимое сопротивление 8 Ом, соединив параллельно два резистора по 16 Ом.

    Пример расчета №2

    Найти общее сопротивление R из трех параллельно соединенных резисторов:

    Общее сопротивление R рассчитывается по формуле:

    Этот метод расчета может быть использованы для расчета любого количества отдельных сопротивлений соединенных параллельно.

    Один важный момент, который необходимо запомнить при расчете параллельно соединенных резисторов – это то, что общее сопротивление всегда будет меньше, чем значение наименьшего сопротивления в этой комбинации.

    Как рассчитать сложные схемы соединения резисторов

    Более сложные соединения резисторов могут быть рассчитаны путем систематической группировки резисторов. На рисунке ниже необходимо посчитать общее сопротивление цепи, состоящей из трех резисторов:


    Для простоты расчета, сначала сгруппируем резисторы по параллельному и последовательному типу соединения.
    Резисторы R2 и R3 соединены последовательно (группа 2). Они в свою очередь соединены параллельно с резистором R1 (группа 1).

    Последовательное соединение резисторов группы 2 вычисляется как сумма сопротивлений R2 и R3:

    В результате мы упрощаем схему в виде двух параллельных резисторов. Теперь общее сопротивление всей схемы можно посчитать следующим образом:

    Расчет более сложных соединений резисторов можно выполнить используя законы Кирхгофа.

    Ток, протекающий в цепи параллельно соединенных резисторах

    Общий ток I протекающий в цепи параллельных резисторов равняется сумме отдельных токов, протекающих во всех параллельных ветвях, причем ток в отдельно взятой ветви не обязательно должен быть равен току в соседних ветвях.

    Несмотря на параллельное соединение, к каждому резистору приложено одно и то же напряжение. А поскольку величина сопротивлений в параллельной цепи может быть разной, то и величина протекающего тока через каждый резистор тоже будет отличаться (по определению закона Ома).

    Рассмотрим это на примере двух параллельно соединенных резисторов. Ток, который течет через каждый из резисторов (I1 и I2) будет отличаться друг от друга поскольку сопротивления резисторов R1 и R2 не равны.
    Однако мы знаем, что ток, который поступает в цепь в точке «А» должен выйти из цепи в точке «B» .

    Первое правило Кирхгофа гласит: «Общий ток, выходящий из цепи равен току входящий в цепь».

    Таким образом, протекающий общий ток в цепи можно определить как:

    Затем с помощью закона Ома можно вычислить ток, который протекает через каждый резистор:

    Ток, протекающий в R1 = U ÷ R1 = 12 ÷ 22 кОм = 0,545 мА

    Ток, протекающий в R 2 = U ÷ R2 = 12 ÷ 47 кОм = 0,255 мА

    Таким образом, общий ток будет равен:

    I = 0,545 мА + 0,255 мА = 0,8 мА

    Это также можно проверить, используя закон Ома:

    I = U ÷ R = 12 В ÷ 15 кОм = 0,8 мА (то же самое)

    где 15кОм — это общее сопротивление двух параллельно соединенных резисторов (22 кОм и 47 кОм)

    И в завершении хочется отметить, что большинство современных резисторов маркируются цветными полосками и назначение ее можно узнать .

    Параллельное соединение резисторов — онлайн калькулятор

    Чтобы быстро вычислить общее сопротивление двух и более резисторов, соединенных параллельно, вы можете воспользоваться следующим онлайн калькулятором:

    Подведем итог

    Когда два или более резистора соединены так, что оба вывода одного резистора соединены с соответствующими выводами другого резистора или резисторов, то говорят, что они соединены между собой параллельно. Напряжение на каждом резисторе внутри параллельной комбинации одинаковое, но токи, протекающие через них, могут отличаться друг от друга, в зависимости от величины сопротивлений каждого резистора.

    Эквивалентное или полное сопротивление параллельной комбинации всегда будет меньше минимального сопротивления резистора входящего в параллельное соединение.

    1 мОм = 0,001 Ом. 1 кОм = 1 000 = 10³ Ом. 1 МОм = 1 000 000 = 10⁶ Ом.

    Эквивалентное сопротивление R eq группы параллельно соединенных резисторов является величиной, обратной сумме величин, обратно пропорциональных сопротивлениям этих резисторов.

    Иными словами, проводимость G параллельно соединенных резисторов равна сумме проводимостей этих резисторов:

    Эта формула для R eq и используется в данном калькуляторе для расчетов. Например, общее сопротивление трех резисторов 10, 15 и 20 ом, соединенных параллельно, равно 4.62 Ом:

    Если параллельно соединены только два резистора, формула упрощается:

    Если имеется n соединенных параллельно одинаковых резисторов R , то их эквивалентное сопротивление будет равно

    Отметим, что общее сопротивление группы из любого количества соединенных параллельно резисторов всегда будет меньше, чем наименьшее сопротивление резистора в группе и добавление нового резистора всегда приведет к уменьшению эквивалентного сопротивления.

    Отметим также, что все резисторы, соединенные параллельно находятся под одним и тем же напряжением. Однако токи, протекающие через отдельные резисторы, отличаются и зависят от их сопротивления. Общий ток через группу резисторов равен сумме токов в отдельных резисторах.

    При соединении нескольких резисторов параллельно всегда нужно учитывать их допуски и рассеиваемую мощность.

    Примеры применения параллельного соединения резисторов

    Одним из примеров параллельного соединения резисторов является шунт в приборе для измерения токов, которые слишком велики для того, чтобы быть напрямую измеренными прибором, предназначенным для измерения небольших токов или напряжений. Для измерения тока параллельно гальванометру или электронному прибору, измеряющему напряжение, подключается резистор с очень маленьким точно известным сопротивлением, изготовленный из материала со стабильными характеристиками. Этот резистор называется шунтом. Измеряемый ток протекает через шунт. В результате на нем падает небольшое напряжение, которое и измеряется вольтметром. Поскольку падение напряжения пропорционально току, протекающему через шунт с известным и точным сопротивлением, вольтметр, подключенный параллельно шунту, можно проградуировать непосредственно в единицах тока (амперах).

    Параллельные и последовательные схемы часто используются для получения точного сопротивления или если резистора с требуемым сопротивлением нет или он слишком дорог, если его приобретать в небольших количествах для массового производства . Например, если устройство содержит много резисторов по 20 кОм и необходим только один резистор 10 кОм. Конечно, несложно найти резистор на 10 кОм. Однако для массового производства иногда бывает лучше поставить два резистора на 20 кОм параллельно, чтобы получить необходимые 10 кОм. Это приведет к снижению себестоимости печатной платы, так как будет снижена оптовая цена компонентов, а также стоимость монтажа, так как будет уменьшено количество типоразмеров элементов, которые должен установить на плату автомат установки компонентов.

    Проверим справедливость показанных здесь формул на простом эксперименте.

    Возьмём два резистора МЛТ-2 на 3 и 47 Ом и соединим их последовательно. Затем измерим общее сопротивление получившейся цепи цифровым мультиметром. Как видим оно равно сумме сопротивлений резисторов, входящих в эту цепочку.


    Замер общего сопротивления при последовательном соединении

    Теперь соединим наши резисторы параллельно и замерим их общее сопротивление.


    Измерение сопротивления при параллельном соединении

    Как видим, результирующее сопротивление (2,9 Ом) меньше самого меньшего (3 Ом), входящего в цепочку. Отсюда вытекает ещё одно известное правило, которое можно применять на практике:

    При параллельном соединении резисторов общее сопротивление цепи будет меньше наименьшего сопротивления, входящего в эту цепь.

    Что ещё нужно учитывать при соединении резисторов?

    Во-первых, обязательно учитывается их номинальная мощность. Например, нам нужно подобрать замену резистору на 100 Ом и мощностью 1 Вт . Возьмём два резистора по 50 Ом каждый и соединим их последовательно. На какую мощность рассеяния должны быть рассчитаны эти два резистора?

    Поскольку через последовательно соединённые резисторы течёт один и тот же постоянный ток (допустим 0,1 А ), а сопротивление каждого из них равно 50 Ом , тогда мощность рассеивания каждого из них должна быть не менее 0,5 Вт . В результате на каждом из них выделится по 0,5 Вт мощности. В сумме это и будет тот самый 1 Вт .

    Данный пример достаточно грубоват. Поэтому, если есть сомнения, стоит брать резисторы с запасом по мощности.

    Подробнее о мощности рассеивания резистора читайте .

    Во-вторых, при соединении стоит использовать однотипные резисторы, например, серии МЛТ. Конечно, нет ничего плохого в том, чтобы брать разные. Это лишь рекомендация.

    Параллельное соединение резисторов, наряду с последовательным, является основным способом соединения элементов в электрической цепи. Во втором варианте все элементы установлены последовательно: конец одного элемента соединен с началом следующего. В такой схеме сила тока на всех элементах одинаковая, а падение напряжений зависит от сопротивления каждого элемента. В последовательном соединении есть два узла. К одному подсоединены начала всех элементов, а ко второму их концы. Условно для постоянного тока можно обозначить их как плюс и минус, а для переменного как фазу и ноль. Благодаря своим особенностям находит широкое применение в электрических схемах, в том числе и со смешанным соединением. Свойства одинаковы для постоянного и переменного тока.

    Расчет общего сопротивления при параллельном соединении резисторов

    В отличие от последовательного соединения, где для нахождения общего сопротивления достаточно сложить значение каждого элемента, для параллельного то же самое будет справедливо для проводимости. А так как она обратно пропорциональна сопротивлению, получим формулу, представленную вместе со схемой на следующем рисунке:

    Необходимо отметить одну важную особенность расчета параллельного соединения резисторов: общее значение будет всегда меньше, чем самое маленькое из них. Для резисторов справедливо как для постоянного, так и для переменного тока. Катушки и конденсаторы имеют свои особенности.

    Сила тока и напряжение

    При расчете параллельного сопротивления резисторов необходимо знать, как рассчитать напряжение и силу тока. В этом случае нам поможет закон Ома, определяющий связь между сопротивлением, силой тока и напряжением.

    Исходя из первой формулировки закона Кирхгофа, получим, что сумма сходящихся в одном узле токов равна нулю. Направление выбираем по направлению протекания тока. Таким образом, положительным направлением для первого узла можно считать входящий ток от источника питания. А отрицательными будут отходящие из каждого резистора. Для второго узла картина противоположна. Исходя из формулировки закона, получим, что суммарный ток равен сумме токов, проходящих через каждый параллельно соединенный резистор.

    Итоговое напряжение же определяется по второму закону Кирхгофа. Оно одинаково для каждого резистора и равно общему. Эта особенность используется для подключения розеток и освещения в квартирах.

    Пример расчета

    В качестве первого примера приведем расчет сопротивления при параллельном соединении одинаковых резисторов. Сила тока, протекающая через них, будет одинаковой. Пример расчета сопротивления выглядит так:

    По этому примеру прекрасно видно, что общее сопротивление ниже в два раза, чем каждое из них. Это соответствует тому, что суммарная сила тока в два раза выше, чем у одного. А также прекрасно соотносится с увеличением проводимости в два раза.

    Второй пример

    Рассмотрим пример параллельного соединения трех резисторов. Для расчета используем стандартную формулу:

    Похожим образом рассчитываются схемы с большим количеством параллельно соединенных резисторов.

    Пример смешанного соединения

    Для смешанного соединения, например, представленного ниже, расчет будет производиться в несколько этапов.

    Для начала последовательные элементы можно условно заменить одним резистором, обладающим сопротивлением, равным сумме двух заменяемых. Далее общее сопротивление считаем тем же способом, что и для предыдущего примера. Данный метод подойдет и для других более сложных схем. Последовательно упрощая схему, можно получить необходимое значение.

    Например, если вместо резистора R3 будут подключены два параллельных, потребуется сначала рассчитать их сопротивление, заменив их эквивалентным. А далее то же самое, что и в примере выше.

    Применение параллельной схемы

    Параллельное соединение резисторов находит свое применение во многих случаях. Последовательное подключение увеличивает сопротивление, а для нашего случая оно уменьшится. Например, для электрической цепи требуется сопротивление в 5 Ом, но есть только резисторы на 10 Ом и выше. Из первого примера мы знаем, что можно получить в два раза меньшее значение сопротивления, если установить два одинаковых резистора параллельно друг другу.

    Уменьшить сопротивление можно еще больше, например, если две пары параллельно соединенных резисторов соединить параллельно относительно друг друга. Можно уменьшить сопротивление еще в два раза, если резисторы имеют одинаковое сопротивление. Комбинируя с последовательным соединением, можно получить любое значение.

    Второй пример — это использование параллельного подключения для освещения и розеток в квартирах. Благодаря такому подключению напряжение на каждом элементе не будет зависеть от их количества и будет одинаковым.

    Еще один пример использования параллельного подключения — это защитное заземление электрооборудования. Например, если человек касается металлического корпуса прибора, на который произойдет пробой, получится параллельное соединения его и защитного проводника. Первым узлом будет место прикосновения, а вторым нулевая точка трансформатора. По проводнику и человеку будет течь разный ток. Величину сопротивления последнего принимают за 1000 Ом, хотя реальное значение зачастую гораздо больше. Если бы не было заземления, весь ток, протекающий в схеме, пошел бы через человека, так как он был бы единственным проводником.

    Параллельное соединение может использоваться и для батарей. Напряжение при этом остается прежним, однако в два раза возрастает их емкость.

    Итог

    При подключении резисторов параллельно, напряжение на них будет одинаковым, а ток равен сумме протекающих через каждый резистор. Проводимость будет ровняться сумме каждого. От этого и получается необычная формула суммарного сопротивления резисторов.

    Необходимо учитывать при расчете параллельного соединения резисторов то, что итоговое сопротивление будет всегда меньше самого маленького. Это также можно объяснить суммированием проводимости резисторов. Последняя будет возрастать при добавлении новых элементов, соответственно и проводимость будет уменьшаться.

    Расчет общего сопротивления цепи при смешанном соединении

    Резистор представляет собой устройство, обладающее устойчивым, стабильным значением сопротивления. Это позволяет выполнять регулировку параметров на любых участках электрической цепи. Существуют различные виды соединений, в том числе и смешанное соединение резисторов. От использования того или иного способа в конкретной схеме, напрямую зависит падение напряжений и распределение токов в цепи. Вариант смешанного соединения состоит из последовательного и параллельного подключения активных сопротивлений. Поэтому в первую очередь нужно рассматривать эти два вида соединений, чтобы понять, как работают другие схемы.

    Последовательное соединение

    Последовательная схема подключения предполагает расположение резисторов в схеме таким образом, что конец первого элемента соединяется с началом второго, а конец второго – с началом третьего и т.д. То есть все резисторы поочередно следуют друг за другом. Сила тока при последовательном соединении будет одинаковой в каждом элементе. В виде формулы это выглядит следующим образом: Iобщ = I1 = I2, где Iобщ является общим током цепи, I1 и I2 – соответствуют токам 1-го и 2-го резистора.

    В соответствии с законом Ома, напряжение источника питания будет равно сумме падений напряжения на каждом резисторе: Uобщ = U1 + U2 = I1r1 + I2r2, в которой Uобщ – напряжение источника электроэнергии или самой сети; U1 и U2 – значение падений напряжения на 1-м и 2-м резисторах; r1 и r2 – сопротивления 1-го и 2-го резисторов. Поскольку токи на любом участке цепи имеют одинаковое значение, формула приобретает вид: Uобщ = I(r1 + r2).

    Таким образом, можно сделать вывод, что при последовательной схеме включения резисторов, электрический ток, протекающий через каждый из них равен общему значению тока во всей цепи. Напряжение на каждом резисторе будет разное, однако их общая сумма составит значение, равное общему напряжению всей электрической цепи. Общее сопротивление цепи также будет равно сумме сопротивлений каждого резистора, включенного в эту цепь.

    Параметры цепи при параллельном соединении

    Параллельное соединение представляет собой включение начальных выходов двух и более резисторов в единой точке, и концов этих же элементов в другой общей точке. Таким образом, фактически происходит соединение каждого резистора непосредственно с источником электроэнергии.

    В результате, напряжение каждого резистора будет одинаковым с общим напряжением цепи: Uобщ = U1 = U2. В свою очередь, значение токов будет разным на каждом резисторе, их распределение становится прямо пропорциональным сопротивлению этих резисторов. То есть, при увеличении сопротивления, сила тока уменьшается, а общий ток становится равен сумме токов, проходящих через каждый элемент. Формула для данного положения выглядит следующим образом: Iобщ= I1 + I2.

    Для расчетов общего сопротивления используется формула: . Она используется при наличии в цепи только двух сопротивлений. В тех случаях, когда сопротивлений в цепи подключено три и более, применяется другая формула:

    Таким образом, значение общего сопротивления электрической цепи будет меньше, чем самое минимальное сопротивление одного из резисторов, подключенных параллельно в эту цепь. На каждый элемент поступает напряжение, одинаковое с напряжением источника электроэнергии. Распределение тока будет прямо пропорциональным сопротивлению резисторов. Значение общего сопротивления резисторов, соединенных параллельно, не должно превышать минимального сопротивления какого-либо элемента.

    Схема смешанного соединения резисторов

    Схема смешанного соединения обладает свойствами схем последовательного и параллельного соединения резисторов. В этом случае элементы частично подключаются последовательно, а другая часть соединяется параллельно. На представленной схеме резисторы R1 и R2 включены последовательно, а резистор R3 соединен параллельно с ними. В свою очередь резистор R4 включается последовательно с предыдущей группой резисторов R1, R2 и R3.

    Расчет сопротивления для такой цепи сопряжен с определенными трудностями. Для того чтобы правильно выполнить расчеты используется метод преобразования. Он заключается в последовательном преобразовании сложной цепи в простейшую цепь за несколько этапов.

    Если для примера вновь использовать представленную схему, то в самом начале определяется сопротивление R12 резисторов R1 и R2, включенных последовательно: R12 = R1 + R2. Далее, нужно определить сопротивление резисторов R123, включенных параллельно, по следующей формуле: R123=R12R3/(R12+R3) = (R1+R2)R3/(R1+R2+R3). На последнем этапе выполняется расчет эквивалентного сопротивления всей цепи, путем суммирования полученных данных R123 и сопротивления R4, включенного последовательно с ним: Rэк = R123 + R4 = (R1 + R2) R3 / (R1 + R2 + R3) + R4.

    В заключение следует отметить, что смешанное соединение резисторов обладает положительными и отрицательными качествами последовательного и параллельного соединения. Это свойство успешно используется на практике в электрических схемах.

    Эквивалентное сопротивление цепи.

    R12 = R1 + R2R = R1 + R23
    U = U3 = I·R = I3·R3U = I· R; U1 = I1·R1
    U2 = U3 = I2·R2 = I3·R3
    U1 = I1·R1 U2 = I2·R2

    Пример 1.1. Цепь постоянного тока со смешанным соединением состоит из четырёх резисторов. Мощность электрической цепи Р = 750 Вт. Определить эквивалентное сопротивление цепи, токи и напряжения на всех резисторах и для всей цепи. Решение проверить, используя баланс мощностей.

    (Указание: номера токов, напряжений и мощностей должны совпадать номерами сопротивлений.)

    Дано: R1=10 Ом; R2 = 50 Ом; R3 = 40 Ом; R4= 6 Ом; Р = 750 Вт.

    Определим эквивалентное сопротивление цепи методом свёртывания.

    Если между сопротивлениями нет узла, то они соединены последовательно, а между двумя узлами имеется параллельное соединение сопротивлений.

    R1 и R2 соединены последовательно, R12 и R3 параллельно, а R123 и R4 последовательно.

    R = R123 + R4 = 24 + 6 = 30 Ом.

    Определим токи и напряжения на всех резисторах.

    Ток и напряжение для всей цепи:

    Рис. 1.9. получается свёртыванием рис. 1.8. На рисунке 1.10 покажем токи и напряжения на резисторах R123 и R4:

    Решение проверим, используя 2-ой закон Кирхгофа.

    U = U3 + U4 = 120 + 30 = 150 В.

    Резистор R123 получается от параллельного соединения резисторов R12 и R3.

    Из рис. 1.11. имеем:

    Решение проверим, используя 1-ый закон Кирхгофа.

    Резистор R12 получается от последовательного соединения резисторов

    Решение проверим, используя 2 — ой закон Кирхгофа.

    150 · 5 =20 · 2 + 100 · 2 + 120 · 3 + 30 · 5 = 40 + 200 + 360 + 150 = 750;

    Ответ представим в виде таблицы:

    R1R2R3R4вся цепь
    R (Ом)
    U (В)
    I (А)
    Р (Вт)

    Задача 1.1. Цепь постоянного тока со смешанным соединением состоит из четырёх резисторов. Дана одна из величин U,I или Р. Определить эквивалент- ное сопротивление цепи, токи и напряжения на всех резисторах и для всей цепи. Решение проверить, используя баланс мощностей.

    (Указание: номера токов, напряжений и мощностей должны совпадать номерами сопротивлений.) Данные выбрать из таблицы 1.1.

    № варианта№ рис.R1 ОмR2 ОмR3 ОмR4 ОмU, I, P
    1.13 1.14 1.15U =120 В I = 5 A P = 1152 Вт
    1.16 1.17 1.18U =160 В I = 10 A P = 576 Вт
    1.19 1.20 1.21U =12 В I = 6 A P = 450 Вт
    1.22 1.13 1.14U =96 В I = 15 A P = 250 Вт
    1.15 1.16 1.17U =48 В I = 15 A P = 375 Вт
    1.18 1.19 1.20U =60 В I = 6 A P = 720 Вт
    1.21 1.22 1.13U =60 В I = 3 A P = 937,5 Вт
    1.14 1.15 1.16U =60 В I = 12 A P = 1440 Вт
    1.17 1.18 1.19U =90 В I = 24A P = 1440 Вт
    1.20 1.21 1.22U =100 В I = 5 A P = 320 Вт
    1.13 1.14 1.15U = 48 В I = 9 A P = 172 Вт
    1.16 1.17 1.18U = 120B I = 10A U = 96 B
    1.19 1.20 1.21U = 90 B I = 9 A U = 90 B
    1.22 1.13 1.14U = 78 B Р = 720 Вт U = 144 Вт
    1.15 1.16 1.17U = 72 Вт I = 12 А U = 90 B
    1.18 1.19 1.20I = 48 A I = 9 A U = 117В
    1.21 1.22I = 15A I = 9A

    Пример 1.2.Дана электрическая цепь со смешанным соединением резисторов. Номера токов, напряжений и мощностей совпадают с номером резистора.

    1.эквивалентное сопротивление электрической цепи;

    2.используя известную величину тока, напряжения или мощности вычис-

    лить токи и напряжения, по закону Ома для участка цепи, на всех резис-

    торах и для всей цепи; законы Кирхгофа использовать для проверки;

    3.проверить решение методом баланса мощностей.

    Рис.R1 ОмR2 ОмR3 ОмR4 ОмR5 ОмR6 ОмДополнительный параметр
    1.23Р = 250 Вт

    Для определения эквивалентного сопротивления используем метод свёртывания.

    R5 и R6 cоединены параллельно, а R4 и R56 последовательно.

    R3 и R456 cоединены параллельно, R1 и R2 c R3456 последовательно.

    Определим ток и напряжение всей цепи:

    Из рис. 1.26 видим, что резисторы R1, R3456, R2 соединены последовательно,

    определим напряжения и токи на R1, R3456, R2. На рис.1.27 покажем токи и напряжения.

    Проверим используя 2-ой закон Кирхгофа.

    U = U1 + U3 + U2 ; 50 = 5 + 30 + 15 = 50

    Рассмотрим резистор R3456. Выделим из рис. 1.25. часть с резисторами R3 и R456, получим рис.1.28. Ток I456 равен I4 т.е. I456 = I4. Определим токи I4 и I1. Из рис. 1.28 видно: напряжение U456 = U3 т.к. R3 и R456 соединены параллельно. Токи в ветвях:

    Проверим, используя 1-ый закон Кирхгофа.

    Отделим из рис.1.24 резисторы R4 и R56. Эти резисторы соединены последовательно. На рис. 1.29 покажем напряжения U4 и U56 = U5 = U6.

    Проверим используя 2-ой закон Кирхгофа.

    Из рис.1.23 видим, что резисторы R5 = R6 соединены параллельно. На рис.1.30 покажем токи I5 и I6. Определим токи на резисторах R5 и R6 .

    Проверим используя 1-ый закон Кирхгофа: I4= I5 + I6; 3 = 2 + 1.

    Из рис.1.30 определим напряжения на R1 и R2: I = I1 = I2 = I3456 = 5 А.

    Решение проверим используя баланс мощностей:

    50·5 = 5·5 + 15·5 + 30·2 + 18·3 + 12·2 + 12·2 = 250.

    R1 ОмR2 ОмR3 ОмR4 ОмR5 ОмR6 ОмВся Цепь
    R Ом
    U В
    I А
    P Вт

    Задача 1.2.Дана электрическая цепь со смешанным соединением резисто- ров. Номера токов, напряжений и мощностей совпадают с номером резистора. Данные выбрать из таблицы 1.2.

    Определить используя заданную величину U,I, или Р:

    1.Эквивалентное сопротивление электрической цепи.

    2.Используя известную величину тока, напряжения или мощности вычислить токи и напряжения, по закону Ома для участка цепи, на всех резисторах и для всей цепи. Законы Кирхгофа использовать для проверки.

    3. Решение проверить методом баланса мощностей.

    Электрические цепи, в которых одна часть сопротивлений соединена последовательно, а другая параллельно, называются цепями со смешанным соединением сопротивлений.

    Общих расчетных формул для таких цепей нет, так как число их разновидностей не ограничено.

    Чаще всего расчет подобных схем начинается с определения эквивалентного сопротивления всей цепи, а затем определяются величины токов и падение напряжения на отдельных участках.

    Для определения эквивалентного сопротивления цепи со смешанным соединением потребителей, питающихся от одного источника тока, необходимо прежде всего разбить эту цепь на отдельные участки, состоящие из последовательного и параллельно соединенных сопротивлений. Далее определяют эквивалентные сопротивления для каждого из участков, а затем и для всей цепи в целом.

    Рассмотрим метод решения задач на смешанное соединение сопротивлений на конкретном примере.

    На рисунке представлена схема смешанного соединения сопротивлений. Ее можно разбить на три участка:

    участок АВ — с двумя параллельно соединенными ветвями;

    участок ВС — с последовательно соединенными сопротивлениями;

    участок СD — с тремя параллельными ветвями.

    Кроме того, нижняя ветвь участка АВ представляет в свою очередь цепь, состоящую из двух последовательно соединенных сопротивлений R2 и R3.

    Центральная ветвь участка СD представляет собой смешанное соединение сопротивлений.

    Расчет данной сложной цепи надо начинать с определения Rэкв для нижней ветви участка АВ и центральной ветви участка СD.

    Теперь мы можем упростить первоначальную схему. Она будет иметь следующий вид

    Определим эквивалентные сопротивления каждого из участков:

    После этих вычислений можно продолжить упрощение схемы

    Полученная упрощенная схема, состоящая в данном случае из трех последовательно соединенных сопротивлений, называется по отношению к реальной эквивалентной схемой.

    Определим Rэкв всей цепи как сумму трех последних сопротивлений

    Зная напряжение источника тока, применяя формулу закона Ома, определим ток в не разветвленном участке смешанной цепи

    Определив величину тока, найдем падение напряжения на участках эквивалентной схемы АВ, ВС, CD:

    Теперь можно определить токи в параллельных ветвях участков АВ и СD

    Остается определить величину токов, протекающих через сопротивления R7 и R8. Для этого надо сначала определить падение напряжения на сопротивлениях R7 и R8.

    Определим падение напряжения на сопротивлении R9:

    Падение напряжения на сопротивлении R7,8 определится как разность UCD и U:

    Теперь определим величины токов, протекающих через сопротивления R7 и R8:

    Величина тока. протекающего через сопротивления R4 и R5, равна I — току в неразветвленном участке цепи.

    Итак, при решении задач на смешанное соединение сопротивлений необходимо, постепенно упрощая схему, определить эквивалентное сопротивление всей цепи, а затем. восстанавливая постепенно реальную схему. вычислить падение напряжения и токи в отдельных ветвях.

    формула расчета общего сопротивления. Примеры параллельного соединения проводников

    Резистор — это элемент электрической схемы, который обладает сопротивлением электрическому току. Классифицируют два типа резисторов: постоянные и переменные (подстроечные). При моделировании той или иной электрической схемы, а также при ремонте электронных изделий, возникает необходимость использовать резистор определенного номинала. Хотя и существует множество различных номиналов постоянных резисторов, в данный момент под рукой может не оказаться требуемого, либо резистора с таким номиналом не существует. Чтобы выйти из такой ситуации, можно использовать как последовательное так и параллельное соединение резисторов. О том, как правильно произвести расчет и подбор различных номиналов сопротивлений, будет рассказано в этой статье.

    Последовательное соединение резисторов — это самая элементарная схема сборки радиодеталей, оно применяется для увеличения общего сопротивления цепи. При последовательном соединении, сопротивление используемых резисторов просто складывается, а вот при параллельном соединении необходимо производить расчет по нижеописанным формулам. Параллельное соединение необходимо для снижения результирующего сопротивления, а также для увеличения мощности, несколько параллельно подключенных резисторов имеют большую мощность, чем у одного.

    На фотографии можно увидеть параллельное подключение резисторов.

    Ниже представлена принципиальная схема параллельного соединения резисторов.

    Общее номинальное сопротивление необходимо рассчитывать по следующей схеме:

    R(общ)=1/(1/R1+1/R2+1/R3+1/R n).

    R1, R2, R3 и Rn — параллельно подключенные резисторы.

    Когда параллельное соединение резисторов состоит всего из двух элементов, в таком случае общее номинальное сопротивление можно высчитать по следующей формуле:

    R(общ)=R1*R2/R1+R2.

    R(общ) — общее сопротивление;

    R1, R2 — параллельно подключенные резисторы.

    В радиотехнике существует следующее правило: если параллельное подключение резисторов состоит из элементов одного номинала, то результирующее сопротивление можно высчитать, разделив номинал резистора на количество соединенных резисторов:

    R(общ) — общее сопротивление;

    R — номинал параллельно подключенного резистора;

    N — количество соединенных элементов.

    Важно учитывать, что при параллельном соединении результирующее сопротивление всегда будет ниже, чем сопротивление самого малого по номиналу резистора.

    Приведем практический пример: возьмем три резистора, со следующими значениями номинального сопротивления: 100 Ом, 150 Ом и 30 Ом. Проведем расчет общего сопротивления, по первой формуле:

    R(общ)=1/(1/100+1/150+1/30)=1/(0,01+0,007+0,03)=1/0,047=21,28Ом.

    После расчета формулы мы видим, что параллельное соединение резисторов, состоящее из трех элементов, с наименьшим номиналом 30 Ом, в результате дает общее сопротивление в электрической цепи 21,28 Ом, что ниже наименьшего номинального сопротивления в цепи почти на 30 процентов.

    Параллельное соединение резисторов чаще всего используют в тех случаях, когда необходимо получить сопротивление с большей мощностью. В таком случае необходимо взять резисторы одинаковой мощности и с одинаковым сопротивлением. Результирующая мощность в таком случае рассчитывается путем умножения мощности одного элемента сопротивления на общее количество параллельно подключенных резисторов в цепи.

    Например: пять резисторов с номиналом в 100 Ом и с мощностью 1 Вт в каждом, подключенные параллельно, имеют общее сопротивление 20 Ом и мощность 5 Вт.

    При последовательном подключении тех же резисторов (мощность так же складывается), получим результирующую мощность 5 Вт, общее сопротивление составит 500 Ом.

    Проверим справедливость показанных здесь формул на простом эксперименте.

    Возьмём два резистора МЛТ-2 на 3 и 47 Ом и соединим их последовательно. Затем измерим общее сопротивление получившейся цепи цифровым мультиметром. Как видим оно равно сумме сопротивлений резисторов, входящих в эту цепочку.


    Замер общего сопротивления при последовательном соединении

    Теперь соединим наши резисторы параллельно и замерим их общее сопротивление.


    Измерение сопротивления при параллельном соединении

    Как видим, результирующее сопротивление (2,9 Ом) меньше самого меньшего (3 Ом), входящего в цепочку. Отсюда вытекает ещё одно известное правило, которое можно применять на практике:

    При параллельном соединении резисторов общее сопротивление цепи будет меньше наименьшего сопротивления, входящего в эту цепь.

    Что ещё нужно учитывать при соединении резисторов?

    Во-первых, обязательно учитывается их номинальная мощность. Например, нам нужно подобрать замену резистору на 100 Ом и мощностью 1 Вт . Возьмём два резистора по 50 Ом каждый и соединим их последовательно. На какую мощность рассеяния должны быть рассчитаны эти два резистора?

    Поскольку через последовательно соединённые резисторы течёт один и тот же постоянный ток (допустим 0,1 А ), а сопротивление каждого из них равно 50 Ом , тогда мощность рассеивания каждого из них должна быть не менее 0,5 Вт . В результате на каждом из них выделится по 0,5 Вт мощности. В сумме это и будет тот самый 1 Вт .

    Данный пример достаточно грубоват. Поэтому, если есть сомнения, стоит брать резисторы с запасом по мощности.

    Подробнее о мощности рассеивания резистора читайте .

    Во-вторых, при соединении стоит использовать однотипные резисторы, например, серии МЛТ. Конечно, нет ничего плохого в том, чтобы брать разные. Это лишь рекомендация.

    На практике нередко встречается задача нахождения сопротивления проводников и резисторов при различных способах соединения. В статье рассмотрено, как рассчитывается сопротивление при параллельном соединении проводников и некоторые другие технические вопросы.

    Сопротивление проводника

    Все проводники имеют свойство препятствовать течению электрического тока, его принято называть электрическим сопротивлением R, оно измеряется в омах. Это основное свойство проводниковых материалов.

    Для ведения электротехнических расчётов применяется удельное сопротивление — ρ Ом·м/мм 2 . Все металлы — хорошие проводники, наибольшее применение получили медь и алюминий, гораздо реже применяется железо. Лучший проводник — серебро, оно применяется в электротехнической и электронной промышленности. Широко распространены сплавы с высоким

    При расчёте сопротивления используется известная из школьного курса физики формула:

    R = ρ · l/S, S — площадь сечения; l — длина.

    Если взять два проводника, то их сопротивление при параллельном соединении станет меньше из-за увеличения общего сечения.

    и нагрев проводника

    Для практических расчётов режимов работы проводников применяется понятие плотности тока — δ А/мм 2 , она вычисляется по формуле:

    δ = I/S, I — ток, S — сечение.

    Ток, проходя по проводнику, нагревает его. Чем больше δ, тем сильнее нагревается проводник. Для проводов и кабелей разработаны нормы допустимой плотности, которые приводятся в Для проводников нагревательных устройств существуют свои нормы плотности тока.

    Если плотность δ выше допустимой, может произойти разрушение проводника, например, при перегреве кабеля у него разрушается изоляция.

    Правилами регламентируется производить расчёт проводников на нагрев.

    Способы соединения проводников

    Любой проводник гораздо удобнее изображать на схемах как электрическое сопротивление R, тогда их легко читать и анализировать. Существует всего три способа соединения сопротивлений. Первый способ самый простой — последовательное соединение.

    На фото видно, что полное сопротивление равно: R = R 1 + R 2 + R 3 .

    Второй способ более сложный — параллельное соединение. Расчёт сопротивления при параллельном соединении выполняется поэтапно. Рассчитывается полная проводимость G = 1/R, а затем полное сопротивление R = 1/G.

    Можно поступить и по-другому, прежде рассчитать общее сопротивление при R1 и R2, после этого повторить операцию и найти R.

    Третий способ соединения наиболее сложный — смешанное соединение, то есть присутствуют все рассмотренные варианты. Схема приведена на фото.

    Для расчёта этой схемы её следует упростить, для этого заменяют резисторы R2 и R3 одним R2,3. Получается несложная схема.

    R2,3,4 = R2,3 · R4/(R2,3 + R4).

    Схема становится ещё проще, в ней остаются резисторы, имеющие последовательное соединение. В более сложных ситуациях используется этот же метод преобразования.

    Виды проводников

    В электронной технике, при производстве проводники представляют собою тонкие полоски медной фольги. Ввиду малой длины сопротивление у них незначительно, им во многих случаях можно пренебречь. Для этих проводников сопротивление при параллельном соединении уменьшается вследствие увеличения сечения.

    Большой раздел проводников представляют обмоточные провода. Они выпускаются разных диаметров — от 0,02 до 5,6 миллиметра. Для мощных трансформаторов и электродвигателей выпускаются медные шинки прямоугольного сечения. Иногда при ремонте заменяют провод большого диаметра на несколько параллельно соединённых меньшего размера.

    Особый раздел проводников представляют провода и кабели, промышленность предоставляет широчайший выбор марок для самых различных нужд. Нередко приходится заменять один кабель на несколько, меньшего сечения. Причины этого бывают самые различные, например, кабель сечением 240 мм 2 очень трудно прокладывать по трассе с крутыми изгибами. Его заменяют на 2×120 мм 2 , и проблема решена.

    Расчёт проводов на нагрев

    Проводник нагревается протекающим током, если его температура превысит допустимую, наступает разрушение изоляции. ПУЭ предусматривает расчёт проводников на нагрев, исходными данными для него являются сила тока и условия внешней среды, в которой проложен проводник. По этим данным из таблиц в ПУЭ выбирается рекомендуемое проводника сечение (провода или кабеля).

    На практике встречаются ситуации, когда нагрузка на действующий кабель сильно возросла. Существует два выхода ‒ заменить кабель на другой, это бывает дорого, или параллельно ему проложить ещё один, чтобы разгрузить основной кабель. В этом случае сопротивление проводника при параллельном соединении уменьшается, следовательно падает выделение тепла.

    Чтобы правильно выбрать сечение второго кабеля, пользуются таблицами ПУЭ, важно при этом не ошибиться с определением его рабочего тока. В этой ситуации охлаждение кабелей будет даже лучше, чем у одного. Рекомендуется рассчитать сопротивление при двух кабелей, чтобы точнее определить их тепловыделение.

    Расчёт проводников на потерю напряжения

    При расположении потребителя R н на большом расстоянии L от источника энергии U 1 возникает довольно большое падение напряжения на проводах линии. К потребителю R н поступает напряжение U 2 значительно ниже начального U 1 . Практически в качестве нагрузки выступает различное электрооборудование, подключаемое к линии параллельно.

    Для решения проблемы производят расчет сопротивления при параллельном соединении всего оборудования, так находится сопротивление нагрузки R н. Далее следует определить сопротивление проводов линии.

    R л = ρ · 2L/S,

    Здесь S — сечение провода линии, мм 2 .

    Параллельное соединение резисторов. При параллельном соединении резисторов нескольких приемников они включаются между двумя точками электрической цепи, образуя параллельные ветви (рис. 26, а). Заменяя

    лампы резисторами с сопротивлениями R1, R2, R3, получим схему, показанную на рис. 26, б.
    При параллельном соединении ко всем резисторам приложено одинаковое напряжение U. Поэтому согласно закону Ома:

    I 1 =U/R 1 ; I 2 =U/R 2 ; I 3 =U/R 3 .

    Ток в неразветвленной части цепи согласно первому закону Кирхгофа I = I 1 +I 2 +I 3 , или

    I = U / R 1 + U / R 2 + U / R 3 = U (1/R 1 + 1/R 2 + 1/R 3) = U / R эк (23)

    Следовательно, эквивалентное сопротивление рассматриваемой цепи при параллельном соединении трех резисторов определяется формулой

    1/R эк = 1/R 1 + 1/R 2 + 1/R 3 (24)

    Вводя в формулу (24) вместо значений 1/R эк, 1/R 1 , 1/R 2 и 1/R 3 соответствующие проводимости G эк, G 1 , G 2 и G 3 , получим: эквивалентная проводимость параллельной цепи равна сумме проводимостей параллельно соединенных резисторов :

    G эк = G 1 + G 2 +G 3 (25)

    Таким образом, при увеличении числа параллельно включаемых резисторов результирующая проводимость электрической цепи увеличивается, а результирующее сопротивление уменьшается.
    Из приведенных формул следует, что токи распределяются между параллельными ветвями обратно пропорционально их электрическим сопротивлениям или прямо пропорционально их проводимостям. Например, при трех ветвях

    I 1: I 2: I 3 = 1/R 1: 1/R 2: 1/R 3 = G 1 + G 2 + G 3 (26)

    В этом отношении имеет место полная аналогия между распределением токов по отдельным ветвям и распределением потоков воды по трубам.
    Приведенные формулы дают возможность определить эквивалентное сопротивление цепи для различных конкретных случаев. Например, при двух параллельно включенных резисторах результирующее сопротивление цепи

    R эк =R 1 R 2 /(R 1 +R 2)

    при трех параллельно включенных резисторах

    R эк =R 1 R 2 R 3 /(R 1 R 2 +R 2 R 3 +R 1 R 3)

    При параллельном соединении нескольких, например n, резисторов с одинаковым сопротивлением R1 результирующее сопротивление цепи Rэк будет в n раз меньше сопротивления R1, т.е.

    R эк = R1 / n (27)

    Проходящий по каждой ветви ток I1, в этом случае будет в п раз меньше общего тока:

    I1 = I / n (28)

    При параллельном соединении приемников, все они находятся под одним и тем же напряжением, и режим работы каждого из них не зависит от остальных. Это означает, что ток, проходящий по какому-либо из приемников, не будет оказывать существенного влияния на другие приемники. При всяком выключении или выходе из строя любого приемника остальные приемники остаются включенными. Поэтому параллельное соединение имеет существенные преимущества перед последовательным, вследствие чего оно получило наиболее широкое распространение. В частности, электрические лампы и двигатели, предназначенные для работы при определенном (номинальном) напряжении, всегда включают параллельно.
    На электровозах постоянного тока и некоторых тепловозах тяговые двигатели в процессе регулирования скорости движения нужно включать под различные напряжения, поэтому они в процессе разгона переключаются с последовательного соединения на параллельное.

    Возьмем три постоянных сопротивления R1, R2 и R3 и включим их в цепь так, чтобы конец первого сопротивления R1 был соединен с началом второго сопротивления R 2, конец второго — с началом третьего R 3, а к началу первого сопротивления и к концу третьего подведем проводники от источника тока (рис. 1 ).

    Такое соединение сопротивлений называется последовательным. Очевидно, что ток в такой цепи будет во всех ее точках один и тот же.

    Рис 1

    Как определить общее сопротивление цепи, если все включенные в нее последовательно сопротивления мы уже знаем? Используя положение, что напряжение U на зажимах источника тока равно сумме падений напряжений на участках цепи, мы можем написать:

    U = U1 + U2 + U3

    где

    U1 = IR1 U2 = IR2 и U3 = IR3

    или

    IR = IR1 + IR2 + IR3

    Вынеся в правой части равенства I за скобки, получим IR = I(R1 + R2 + R3) .

    Поделив теперь обе части равенства на I , будем окончательно иметь R = R1 + R2 + R3

    Таким образом, мы пришли к выводу, что при последовательном соединении сопротивлений общее сопротивление всей цепи равно сумме сопротивлений отдельных участков.

    Проверим этот вывод на следующем примере. Возьмем три постоянных сопротивления, величины которых известны (например, R1 == 10 Ом, R 2 = 20 Ом и R 3 = 50 Ом). Соединим их последовательно (рис. 2 ) и подключим к источнику тока, ЭДС которого равна 60 В ( пренебрегаем).


    Рис. 2. Пример последовательного соединения трех сопротивлений

    Подсчитаем, какие показания должны дать приборы, включенные, как показано на схеме, если замкнуть цепь. Определим внешнее сопротивление цепи: R = 10 + 20 + 50 = 80 Ом.

    Найдем ток в цепи : 60 / 80 = 0 ,75 А

    Зная ток в цепи и сопротивления ее участков, определим падение напряжения на каждое участке цепи U 1 = 0,75х 10 = 7,5 В, U 2 = 0,75 х 20=15 В, U3 = 0,75 х 50 = 37,5 В.

    Зная падение напряжений на участках, определим общее падение напряжения во внешней цепи, т. е. напряжение на зажимах источника тока U = 7,5+15 + 37,5 = 60 В.

    Мы получили таким образом, что U = 60 В, т. е. несуществующее равенство ЭДС источника тока и его напряжения. Объясняется это тем, что мы пренебрегли внутренним сопротивлением источника тока.

    Замкнув теперь ключ выключатель К, можно убедиться по приборам, что наши подсчеты примерно верны.

    Возьмем два постоянных сопротивления R1 и R2 и соединим их так, чтобы начала этих сопротивлений были включены в одну общую точку а, а концы — в другую общую точку б. Соединив затем точки а и б с источником тока, получим замкнутую электрическую цепь. Такое соединение сопротивлений называется параллельным соединением.


    Рис 3. Параллельное соединение сопротивлений

    Проследим течение тока в этой цепи. От положительного полюса источника тока по соединительному проводнику ток дойдет до точки а. В точке а он разветвится, так как здесь сама цепь разветвляется на две отдельные ветви: первую ветвь с сопротивлением R1 и вторую — с сопротивлением R2. Обозначим токи в этих ветвях соответственно через I1 и I 2. Каждый из этих токов пойдет по своей ветви до точки б. В этой точке произойдет слияние токов в один общий ток, который и придет к отрицательному полюсу источника тока.

    Таким образом, при параллельном соединении сопротивлений получается разветвленная цепь. Посмотрим, какое же будет соотношение между токами в составленной нами цепи.

    Включим амперметр между положительным полюсом источника тока (+) и точкой а и заметим его показания. Включив затем амперметр (показанный «а рисунке пунктиром) в провод, соединяющий точку б с отрицательным полюсом источника тока (-), заметим, что прибор покажет ту же величину силы тока.

    Значит, до ее разветвления (до точки а) равна силе тока после разветвления цепи (после точки б).

    Будем теперь включать амперметр поочередно в каждую ветвь цепи, запоминая показания прибора. Пусть в первой ветви амперметр покажет силу тока I1 , а во второй — I 2. Сложив эти два показания амперметра, мы получим суммарный ток, по величине равный току I до разветвления (до точки а).

    Следовательно, сила тока, протекающего до точки разветвления, равна сумме сил токов, утекающих от этой точки. I = I1 + I2 Выражая это формулой, получим

    Это соотношение, имеющее большое практическое значение, носит название закона разветвленной цепи .

    Рассмотрим теперь, каково будет соотношение между токами в ветвях.

    Включим между точками а и б вольтметр и посмотрим, что он нам покажет. Во-первых, вольтметр покажет напряжение источника тока, так как он подключен, как это видно из рис. 3 , непосредственно к зажимам источника тока. Во-вторых, вольтметр покажет падения напряжений U1 и U2 на сопротивлениях R1 и R2, так как он соединен с началом и концом каждого сопротивления.

    Следовательно, при параллельном соединении сопротивлений напряжение на зажимах источника тока равно падению напряжения на каждом сопротивлении.

    Это дает нам право написать, что U = U1 = U2 ,

    где U — напряжение на зажимах источника тока; U1 — падение напряжения на сопротивлении R1 , U2 — падение напряжения на сопротивлении R2. Вспомним, что падение напряжения на участке цепи численно равно произведению силы тока, протекающего через этот участок, на сопротивление участка U = IR .

    Поэтому для каждой ветви можно написать: U1 = I1R1 и U2 = I2R2 , но так как U1 = U2, то и I1R1 = I2R2 .

    Применяя к этому выражению правило пропорции, получим I1/ I2 = U2 / U1 т. е. ток в первой ветви будет во столько раз больше (или меньше) тока во второй ветви, во сколько раз сопротивление первой ветви меньше (или больше) сопротивления второй ветви.

    Итак, мы пришли к важному выводу, заключающемуся в том, что при параллельном соединении сопротивлений общий ток цепи разветвляется на токи, обратно пропорциональные величинам сопротивлении параллельных ветвей. Иначе говоря, чем больше сопротивление ветви, тем меньший ток потечет через нее, и, наоборот, чем меньше сопротивление ветви, тем больший ток потечет через эту ветвь.

    Убедимся в правильности этой зависимости на следующем примере. Соберем схему, состоящую из двух параллельно соединенных сопротивлений R1 и R 2, подключенных к источнику тока. Пусть R1 = 10 Ом, R2 = 20 Ом и U = 3 В.

    Подсчитаем сначала, что покажет нам амперметр, включенный в каждую ветвь:

    I1 = U / R1 = 3 / 10 = 0 ,3 А = 300 мА

    I 2 = U / R 2 = 3 / 20 = 0,15 А = 150 мА

    Общий ток в цепи I = I1 +I2 = 300 + 150 = 450 мА

    Проделанный нами расчет подтверждает, что при параллельном соединении сопротивлений ток в цепи разветвляется обратно пропорционально сопротивлениям.

    Действительно, R1 == 10 Ом вдвое меньше R 2 = 20 Ом, при этом I1 = 300 мА вдвое больше I2 = 150 мА. Общий ток в цепи I = 450 мА разветвился на две части так, что большая его часть (I1 = 300 мА) пошла через меньшее сопротивление (R1 = 10 Ом), а меньшая часть (R2 = 150 мА) -через большее сопротивление (R 2 = 20 Ом).

    Такое разветвление тока в параллельных ветвях сходно с течением жидкости по трубам. Представьте себе трубу А, которая в каком-то месте разветвляется на две трубы Б и В различного диаметра (рис. 4). Так как диаметр трубы Б больше диаметра трубок В, то через трубу Б в одно и то же время пройдет больше воды, чем через трубу В, которая оказывает потоку воды большее сопротивление.

    Рис. 4

    Рассмотрим теперь, чему будет равно общее сопротивление внешней цепи, состоящей из двух параллельно соединенных сопротивлений.

    Под этим общим сопротивлением внешней цепи надо понимать такое сопротивление, которым можно было бы заменить при данном напряжении цепи оба параллельно включенных сопротивления, не изменяя при этом тока до разветвления. Такое сопротивление называется эквивалентным сопротивлением.

    Вернемся к цепи, показанной на рис. 3, и посмотрим, чему будет равно эквивалентное сопротивление двух параллельно соединенных сопротивлений. Применяя к этой цепи закон Ома, мы можем написать: I = U/R , где I — ток во внешней цепи (до точки разветвления), U — напряжение внешней цепи, R — сопротивление внешней цепи, т. е. эквивалентное сопротивление.

    Точно так же для каждой ветви I1 = U1 / R1 , I2 = U2 / R2 , где I1 и I 2 — токи в ветвях; U1 и U2 — напряжение на ветвях; R1 и R2 — сопротивления ветвей.

    По закону разветвленной цепи: I = I1 + I2

    Подставляя значения токов, получим U / R = U1 / R1 + U2 / R2

    Так как при параллельном соединении U = U1 = U2 , то можем написать U / R = U / R1 + U / R2

    Вынеся U в правой части равенства за скобки, получим U / R = U (1 / R1 + 1 / R2 )

    Разделив теперь обе части равенства на U , будем окончательно иметь 1 / R = 1 / R1 + 1 / R2

    Помня, что проводимостью называется величина, обратная сопротивлению , мы можем сказать, что в полученной формуле 1 / R — проводимость внешней цепи; 1 / R1 проводимость первой ветви; 1 / R2- проводимость второй ветви.

    На основании этой формулы делаем вывод: при параллельном соединении проводимость внешней цепи равна сумме проводимостей отдельных ветвей.

    Следовательно, чтобы определить эквивалентное сопротивление включенных параллельно сопротивлений, надо определить проводимость цепи и взять величину, ей обратную.

    Из формулы также следует, что проводимость цепи больше проводимости каждой ветви, а это значит, что эквивалентное сопротивление внешней цепи меньше наименьшего из включенных параллельно сопротивлений.

    Рассматривая случай параллельного соединения сопротивлений, мы взяли наиболее простую цепь, состоящую из двух ветвей. Однако на практике могут встретиться случаи, когда цепь состоит из трех и более параллельных ветвей. Как же поступать в этих случаях?

    Оказывается, все полученные нами соотношения остаются справедливыми и для цепи, состоящей из любого числа параллельно соединенных сопротивлений.

    Чтобы убедиться в этом, рассмотрим следующий пример.

    Возьмем три сопротивления R1 = 10 Ом, R2 = 20 Ом и R3 = 60 Ом и соединим их параллельно. Определим эквивалентное сопротивление цепи (рис. 5 ).


    Рис. 5. Цепь с тремя параллельно соединенными сопротивлениями

    Применяя для этой цепи формулу 1 / R = 1 / R1 + 1 / R2 , можем написать 1 / R = 1 / R1 + 1 / R2 + 1 / R3 и, подставляя известные величины, получим 1 / R = 1 / 10 + 1 / 20 + 1 / 60

    Сложим эта дроби: 1/R = 10 / 60 = 1 / 6, т. е.. проводимость цепи 1 / R = 1 / 6 Следовательно, эквивалентное сопротивление R = 6 Ом.

    Таким образом, эквивалентное сопротивление меньше наименьшего из включенных параллельно в цепь сопротивлений , т. е. меньше сопротивления R1.

    Посмотрим теперь, действительно ли это сопротивление является эквивалентным, т. е. таким, которое могло бы заменить включенные параллельно сопротивления в 10, 20 и 60 Ом, не изменяя при этом силы тока до разветвления цепи.

    Допустим, что напряжение внешней цепи, а следовательно, и напряжение на сопротивлениях R1, R2, R3 равно 12 В. Тогда сила токов в ветвях будет: I1 = U/R1 = 12 / 10 = 1 ,2 А I 2 = U/R 2 = 12 / 20 = 1 ,6 А I 3 = U/R1 = 12 / 60 = 0,2 А

    Общий ток в цепи получим, пользуясь формулой I = I1 + I2 + I3 =1,2 + 0,6 + 0,2 = 2 А.

    Проверим по формуле закона Ома, получится ли в цепи ток силой 2 А, если вместо трех параллельно включенных известных нам сопротивлений включено одно эквивалентное им сопротивление 6 Ом.

    I = U / R = 12 / 6 = 2 А

    Как видим, найденное нами сопротивление R = 6 Ом действительно является для данной цепи эквивалентным.

    В этом можно убедиться и на измерительных приборах, если собрать схему с взятыми нами сопротивлениями, измерить ток во внешней цепи (до разветвления), затем заменить параллельно включенные сопротивления одним сопротивлением 6 Ом и снова измерить ток. Показания амперметра и в том и в другом случае будут примерно одинаковыми.

    На практике могут встретиться также параллельные соединения, для которых рассчитать эквивалентное сопротивление можно проще, т. е. не определяя предварительно проводимостей, сразу найти сопротивление.

    Например, если соединены параллельно два сопротивления R1 и R2 , то формулу 1 / R = 1 / R1 + 1 / R2 можно преобразовать так: 1/R = (R2 + R1) / R1 R2 и, решая равенство относительно R, получить R = R1 х R2 / (R1 + R2 ), т. е. при параллельном соединении двух сопротивлений эквивалентное сопротивление цепи равно произведению включенных параллельно сопротивлений, деленному на их сумму.

    Формула расчёта общего сопротивления при параллельном соединении

    Из закона Ома и первого и второго правил Кирхгофа следует:

    При параллельном соединении величина обратная полному сопротивлению, равна сумме величин, обратных сопротивлений ветвей.

    При параллельном соединении полное сопротивление цепи меньше самого малого из сопротивлений ветвей.

    Поскольку 1/R = G, т.е. проводимость, то
    при параллельном соединении электрические проводимости отдельных ветвей складываются

    Параллельное соединение двух сопротивлений

    При параллельном соединении двух сопротивлений формула (1) упрощается

    Параллельное соединение двух сопротивлений

    При параллельном соединении двух сопротивлений формула (1) упрощается

    Параллельное соединение резисторов — одно из двух видов электрических соединений, когда оба вывода одного резистора соединены с соответствующими выводами другого резистора или резисторов. Зачастую резисторы соединяют последовательно или параллельно для того, чтобы создать более сложные электронные схемы.

    Схема параллельного соединения резисторов показан на рисунке ниже. При параллельном соединении резисторов, напряжение на всех резисторах будет одинаковым, а протекающий через них ток будет пропорционален их сопротивлению:

    Формула параллельного соединения резисторов

    Общее сопротивление нескольких резисторов соединенных параллельно определяется по следующей формуле:

    Ток, протекающий через отдельно взятый резистор, согласно закону Ома, можно найти по формуле:

    Параллельное соединение резисторов — расчет

    Пример №1

    При разработке устройства, возникла необходимость установить резистор с сопротивлением 8 Ом. Если мы просмотрим весь номинальный ряд стандартных значений резисторов, то мы увидим, что резистора с сопротивлением в 8 Ом в нем нет.

    Выходом из данной ситуации будет использование двух параллельно соединенных резисторов. Эквивалентное значение сопротивления для двух резисторов соединенных параллельно рассчитывается следующим образом:

    Данное уравнение показывает, что если R1 равен R2, то сопротивление R составляет половину сопротивления одного из двух резисторов. При R = 8 Ом, R1 и R2 должны, следовательно, иметь значение 2 × 8 = 16 Ом.
    Теперь проведем проверку, рассчитав общее сопротивление двух резисторов:

    Таким образом, мы получили необходимое сопротивление 8 Ом, соединив параллельно два резистора по 16 Ом.

    Пример расчета №2

    Найти общее сопротивление R из трех параллельно соединенных резисторов:

    Общее сопротивление R рассчитывается по формуле:

    Этот метод расчета может быть использованы для расчета любого количества отдельных сопротивлений соединенных параллельно.

    Один важный момент, который необходимо запомнить при расчете параллельно соединенных резисторов – это то, что общее сопротивление всегда будет меньше, чем значение наименьшего сопротивления в этой комбинации.

    Как рассчитать сложные схемы соединения резисторов

    Более сложные соединения резисторов могут быть рассчитаны путем систематической группировки резисторов. На рисунке ниже необходимо посчитать общее сопротивление цепи, состоящей из трех резисторов:


    Для простоты расчета, сначала сгруппируем резисторы по параллельному и последовательному типу соединения.
    Резисторы R2 и R3 соединены последовательно (группа 2). Они в свою очередь соединены параллельно с резистором R1 (группа 1).

    Последовательное соединение резисторов группы 2 вычисляется как сумма сопротивлений R2 и R3:

    В результате мы упрощаем схему в виде двух параллельных резисторов. Теперь общее сопротивление всей схемы можно посчитать следующим образом:

    Расчет более сложных соединений резисторов можно выполнить используя законы Кирхгофа.

    Ток, протекающий в цепи параллельно соединенных резисторах

    Общий ток I протекающий в цепи параллельных резисторов равняется сумме отдельных токов, протекающих во всех параллельных ветвях, причем ток в отдельно взятой ветви не обязательно должен быть равен току в соседних ветвях.

    Несмотря на параллельное соединение, к каждому резистору приложено одно и то же напряжение. А поскольку величина сопротивлений в параллельной цепи может быть разной, то и величина протекающего тока через каждый резистор тоже будет отличаться (по определению закона Ома).

    Рассмотрим это на примере двух параллельно соединенных резисторов. Ток, который течет через каждый из резисторов ( I1 и I2 ) будет отличаться друг от друга поскольку сопротивления резисторов R1 и R2 не равны.
    Однако мы знаем, что ток, который поступает в цепь в точке «А» должен выйти из цепи в точке «B» .

    Первое правило Кирхгофа гласит: «Общий ток, выходящий из цепи равен току входящий в цепь».

    Таким образом, протекающий общий ток в цепи можно определить как:

    Затем с помощью закона Ома можно вычислить ток, который протекает через каждый резистор:

    Ток, протекающий в R1 = U ÷ R1 = 12 ÷ 22 кОм = 0,545 мА

    Ток, протекающий в R 2 = U ÷ R2 = 12 ÷ 47 кОм = 0,255 мА

    Таким образом, общий ток будет равен:

    I = 0,545 мА + 0,255 мА = 0,8 мА

    Это также можно проверить, используя закон Ома:

    I = U ÷ R = 12 В ÷ 15 кОм = 0,8 мА (то же самое)

    где 15кОм — это общее сопротивление двух параллельно соединенных резисторов (22 кОм и 47 кОм)

    И в завершении хочется отметить, что большинство современных резисторов маркируются цветными полосками и назначение ее можно узнать здесь.

    Параллельное соединение резисторов — онлайн калькулятор

    Чтобы быстро вычислить общее сопротивление двух и более резисторов, соединенных параллельно, вы можете воспользоваться следующим онлайн калькулятором:

    Подведем итог

    Когда два или более резистора соединены так, что оба вывода одного резистора соединены с соответствующими выводами другого резистора или резисторов, то говорят, что они соединены между собой параллельно. Напряжение на каждом резисторе внутри параллельной комбинации одинаковое, но токи, протекающие через них, могут отличаться друг от друга, в зависимости от величины сопротивлений каждого резистора.

    Эквивалентное или полное сопротивление параллельной комбинации всегда будет меньше минимального сопротивления резистора входящего в параллельное соединение.

    Как правильно соединять резисторы?

    О том, как соединять конденсаторы и рассчитывать их общую ёмкость уже рассказывалось на страницах сайта. А как соединять резисторы и посчитать их общее сопротивление? Именно об этом и будет рассказано в этой статье.

    Резисторы есть в любой электронной схеме, причём их номинальное сопротивление может отличаться не в 2 – 3 раза, а в десятки и сотни раз. Так в схеме можно найти резистор на 1 Ом, и тут же неподалёку на 1000 Ом (1 кОм)!

    Поэтому при сборке схемы либо ремонте электронного прибора может потребоваться резистор с определённым номинальным сопротивлением, а под рукой такого нет. В результате быстро найти подходящий резистор с нужным номиналом не всегда удаётся. Это обстоятельство тормозит процесс сборки схемы или ремонта. Выходом из такой ситуации может быть применение составного резистора.

    Для того чтобы собрать составной резистор нужно соединить несколько резисторов параллельно или последовательно и тем самым получить нужное нам номинальное сопротивление. На практике это пригождается постоянно. Знания о правильном соединении резисторов и расчёте их общего сопротивления выручают и ремонтников, восстанавливающих неисправную электронику, и радиолюбителей, занятых сборкой своего электронного устройства.

    Последовательное соединение резисторов.

    В жизни последовательное соединение резисторов имеет вид:


    Последовательно соединённые резисторы серии МЛТ

    Принципиальная схема последовательного соединения выглядит так:

    На схеме видно, что мы заменяем один резистор на несколько, общее сопротивление которых равно тому, который нам необходим.

    Подсчитать общее сопротивление при последовательном соединении очень просто. Нужно сложить все номинальные сопротивления резисторов входящих в эту цепь. Взгляните на формулу.

    Общее номинальное сопротивление составного резистора обозначено как Rобщ.

    Номинальные сопротивления резисторов включённых в цепь обозначаются как R1, R2, R3,…RN.

    Применяя последовательное соединение, стоит помнить одно простое правило:

    Из всех резисторов, соединённых последовательно главную роль играет тот, у которого самое большое сопротивление. Именно он в значительной степени влияет на общее сопротивление.

    Так, например, если мы соединяем три резистора, номинал которых равен 1, 10 и 100 Ом, то в результате мы получим составной на 111 Ом. Если убрать резистор на 100 Ом, то общее сопротивление цепочки резко уменьшиться до 11 Ом! А если убрать, к примеру, резистор на 10 Ом, то сопротивление будет уже 101 Ом. Как видим, резисторы с малыми сопротивлениями в последовательной цепи практически не влияют на общее сопротивление.

    Параллельное соединение резисторов.

    Можно соединять резисторы и параллельно:


    Два резистора МЛТ-2, соединённых параллельно

    Принципиальная схема параллельного соединения выглядит следующим образом:

    Для того чтобы подсчитать общее сопротивление нескольких параллельно соединённых резисторов понадобиться знание формулы. Выглядит она вот так:

    Эту формулу можно существенно упростить, если применять только два резистора. В таком случае формула примет вид:

    Есть несколько простых правил, позволяющих без предварительного расчёта узнать, каково должно быть сопротивление двух резисторов, чтобы при их параллельном соединении получить то, которое требуется.

    Если параллельно соединены два резистора с одинаковым сопротивлением, то общее сопротивление этих резисторов будет ровно в два раза меньше, чем сопротивление каждого из резисторов, входящих в эту цепочку.

    Это правило исходит из простой формулы для расчёта общего сопротивления параллельной цепи, состоящей из резисторов одного номинала. Она очень проста. Нужно разделить номинальное сопротивление одного из резисторов на общее их количество:

    Здесь R1 – номинальное сопротивление резистора. N – количество резисторов с одинаковым номинальным сопротивлением.

    Ознакомившись с приведёнными формулами, вы скажите, что все они справедливы для расчёта ёмкости параллельно и последовательно соединённых конденсаторов. Да, только в отношении конденсаторов всё действует с точностью до «наоборот”. Узнать подробнее о соединении конденсаторов можно здесь.

    Проверим справедливость показанных здесь формул на простом эксперименте.

    Возьмём два резистора МЛТ-2 на 3 и 47 Ом и соединим их последовательно. Затем измерим общее сопротивление получившейся цепи цифровым мультиметром. Как видим оно равно сумме сопротивлений резисторов, входящих в эту цепочку.


    Замер общего сопротивления при последовательном соединении

    Теперь соединим наши резисторы параллельно и замерим их общее сопротивление.


    Измерение сопротивления при параллельном соединении

    Как видим, результирующее сопротивление (2,9 Ом) меньше самого меньшего (3 Ом), входящего в цепочку. Отсюда вытекает ещё одно известное правило, которое можно применять на практике:

    При параллельном соединении резисторов общее сопротивление цепи будет меньше наименьшего сопротивления, входящего в эту цепь.

    Что ещё нужно учитывать при соединении резисторов?

    Во-первых, обязательно учитывается их номинальная мощность. Например, нам нужно подобрать замену резистору на 100 Ом и мощностью 1 Вт. Возьмём два резистора по 50 Ом каждый и соединим их последовательно. На какую мощность рассеяния должны быть рассчитаны эти два резистора?

    Поскольку через последовательно соединённые резисторы течёт один и тот же постоянный ток (допустим 0,1 А), а сопротивление каждого из них равно 50 Ом, тогда мощность рассеивания каждого из них должна быть не менее 0,5 Вт. В результате на каждом из них выделится по 0,5 Вт мощности. В сумме это и будет тот самый 1 Вт.

    Данный пример достаточно грубоват. Поэтому, если есть сомнения, стоит брать резисторы с запасом по мощности.

    Подробнее о мощности рассеивания резистора читайте тут.

    Во-вторых, при соединении стоит использовать однотипные резисторы, например, серии МЛТ. Конечно, нет ничего плохого в том, чтобы брать разные. Это лишь рекомендация.

    резисторов, включенных последовательно и параллельно | Комбинации резисторов

    Результаты обучения

    • Рассчитайте общее сопротивление различных комбинаций резисторов, т. Е. Последовательных, параллельных и последовательно-параллельных.
    • Покажите, как резисторы используются в качестве делителей напряжения и тока.
    • Рассчитайте сопротивление и значение мощности для последовательного резистора, понижающего напряжение.

    Отдельные резисторы могут быть соединены вместе в последовательном соединении, параллельном соединении или комбинации как последовательного, так и параллельного соединения.Это приводит к более сложной схеме, полное сопротивление которой представляет собой комбинацию отдельных резисторов.

    Комбинация резисторов серии

    Для подключения резисторов серии , они соединяются встык вместе в одну линию, как показано для Рисунок 1 . Характеристики последовательно соединенных резисторов можно резюмировать следующим образом:

    • Общее сопротивление цепи ( R T ) увеличивается при последовательном включении дополнительных резисторов и уменьшается при удалении резисторов.
    • Чтобы определить общее сопротивление цепи, просто найдите сумму отдельных сопротивлений нагрузок.
    • В этом примере, если резисторы имеют маркировку R 1 , R 2 и R 3 , то общее сопротивление R T рассчитывается по формуле

    Рисунок 1 Резисторы, подключенные последовательно.

    ПРИМЕР 1

    Проблема: Три резистора, R 1 (4 Ом), R 2 (50 Ом) и R 3 (75 Ом) подключены последовательно, как показано на Рисунок 2 .Определите значение общего сопротивления комбинированной цепи.

    Рисунок 2 Схема для примера 1.

    Решение:

    Резисторы, соединенные последовательно, используются в качестве делителей напряжения , , как показано на схеме Рисунок 3 . Делители напряжения широко используются в цепях, где один источник напряжения должен обеспечивать несколько разных значений напряжения для разных частей цепи.

    Характеристики схемы последовательного делителя напряжения можно резюмировать следующим образом:

    • Через каждый резистор проходит одинаковый ток.
    • Входное напряжение делится пропорционально между последовательно соединенными резисторами.
    • Падение напряжения на резисторе в последовательной цепи прямо пропорционально сопротивлению резистора.
    • Чем выше значение сопротивления, тем больше падение напряжения.

    Рисунок 3 Схема делителя напряжения .

    Для схемы делителя напряжения падение напряжения на каждом резисторе обычно является фактором, который необходимо определить.Падение напряжения на любом резисторе пропорционально отношению его сопротивления к общему сопротивлению цепи.

    Формула делителя напряжения позволяет рассчитать падение напряжения на любом из последовательно подключенных резисторов без необходимости сначала рассчитывать значение тока цепи. Заявлено в виде формулы:

    ПРИМЕР 2

    Резисторы R 1 (5 кОм), R 2 (3 кОм) и R 3 (2 кОм) ) соединены последовательно, образуя делитель напряжения, как показано на Рисунок 4 .Если на схему подается входное напряжение 9 вольт, рассчитайте значение падения напряжения на каждом из резисторов, используя формулу делителя напряжения.

    Рисунок 4 Схема для примера 2.

    Решение:

    ПРИМЕР 3

    У вас есть источник на 120 В и вы хотите использовать последовательно понижающий резистор в сочетании с 6 В. Сигнальная лампа @ 150 мА для индикации подачи питания (, рис. 5, ).Определите значение сопротивления падения серии и требуемую мощность.

    Рисунок 5 Схема для примера 3.

    Решение:

    Параллельная комбинация резисторов

    Резисторы соединены параллельно путем соединения бок о бок как показано на Рисунок 6 . Обратите внимание, что два конца резисторов подключены к одним и тем же двум точкам.

    Характеристики параллельно соединенных резисторов можно резюмировать следующим образом:

    • Общее сопротивление (R T ) сформированной цепи на меньше , чем сопротивление самого низкого значения сопротивления, присутствующего в любой из ветвей.
    • Каждый резистор обеспечивает отдельный параллельный путь для прохождения тока.
    • Если у вас есть несколько резисторов одного номинала, подключенных параллельно, то общее сопротивление легче всего найти, разделив общее значение сопротивления на количество подключенных резисторов.Для трех резисторов по 150 Ом, включенных параллельно, общее сопротивление составляет

    Рисунок 6 Резисторы, подключенные параллельно.

    Чтобы найти полное сопротивление двух неравных значений резисторов, соединенных параллельно (очень распространенное использование), используется формула произведения на сумму. Эта формула:

    ПРИМЕР 4

    Проблема: Резистор 60 Ом подключается параллельно с резистором 40 Ом, как показано на Рисунок 7 .Определите значение общего комбинированного сопротивления двух компонентов, используя формулу «произведение на сумму».

    Рисунок 7 Схема для примера 4.

    Решение:

    Формула произведения на сумму лучше всего работает для двух резисторов, включенных параллельно. Если параллельно подключено более двух резисторов, использование становится труднее и менее практичным. Для более чем двух резисторов разных номиналов, соединенных параллельно, используется общая формула для полного сопротивления параллельной цепи.Эта формула:

    ПРИМЕР 5

    Проблема: Три резистора, R 1 (120 Ом), R 2 (60 Ом) и R 3 ( 40 Ом) подключены параллельно, как показано на Рисунок 8 . Определите значение общего сопротивления комбинированной цепи.

    Рисунок 8 Цепь для примера 5.

    Решение:

    Параллельные резистивные цепи можно рассматривать как делители тока , потому что ток разделяется или делится между различными резисторами, как показано на рисунке 9 .

    Характеристики схемы параллельного делителя тока можно резюмировать следующим образом:

    • Ток, протекающий через каждый резистор ответвления, обратно пропорционален его значению сопротивления.
    • Чем меньше значение сопротивления, тем больше ток, и наоборот.
    • Резисторы с одинаковым сопротивлением пропускают через них одинаковый ток.
    • Формула, описывающая делитель тока, аналогична формуле для делителя напряжения и может быть выражена следующим образом:

    Рисунок 9 Схема делителя тока .

    ПРИМЕР 6

    Проблема: Резисторы R 1 R 2 и R 3 (2 Ом, 3 Ом и 6 Ом соответственно) подключены параллельно, как показано на Рисунок 10 . Используйте формулу делителя тока, чтобы вычислить значение тока, протекающего через каждый из нагрузочных резисторов, если общий ток, протекающий в цепи, составляет 10 ампер.

    Рисунок 10 Схема для примера 6.

    Решение:

    ПРИМЕР 7

    Проблема: При параллельном подключении дополнительных нагрузок общее сопротивление цепи уменьшается. Для цепи, показанной на рис. 11 , определите полное сопротивление цепи при каждом из следующих рабочих условий:

    1. Переключатели 1 и 2 замкнуты.
    2. Переключатели 1, 2 и 3 замкнуты.

    Рисунок 11 Схема для примера 7.

    Решение:

    Последовательно-параллельная комбинация резисторов

    Комбинированные резистивные цепи, также известные как последовательно-параллельные резистивные цепи , объединяют резисторы последовательно с резисторами, включенными параллельно, как показано на рисунке 12 .

    Правила, регулирующие эти цепи, такие же, как и правила, разработанные для последовательных и параллельных цепей. Сначала определяется сопротивление совокупного полного сопротивления параллельной части.Затем общее сопротивление параллельной части добавляется к любому последовательному сопротивлению, чтобы найти общее сопротивление последовательно-параллельной комбинированной цепи.

    Рисунок 12 Последовательно-параллельное соединение резисторов.

    ПРИМЕР 8

    Проблема: Резистор 9 Ом, R 1 , и резистор 60 Ом, R 2 , подключены параллельно друг другу и последовательно с резистор 40 Ом, R 3 , как показано на Рисунок 13 .Определите общее сопротивление этой последовательно-параллельной комбинации резисторов.

    Рисунок 13 Схема для примера 8.

    Решение:

    ПРИМЕР 9

    Проблема: Показания сопротивления можно использовать для проверки схем на наличие неисправностей. Как определено в предыдущем примере, нормальное полное сопротивление этой последовательно-параллельной схемы , рис. 14, составляет 60 Ом.

    1. Найдите новое значение R T , если резистор R 1 будет поврежден разомкнут , в то время как значения сопротивления R 2 и R 3 останутся равными тем же.
    2. Аналогичным образом найдите новое значение R T , если резистор R 3 будет поврежден закорочен , в то время как значения сопротивления R 1 и R 2 останутся такой же.

    Рисунок 14 Цепь для примера 9.

    Решение:

    1. При разомкнутом отказе R 1 цепь будет состоять из 3 R последовательно с 2 R и всего сопротивление будет:

    1. При замкнутом коротком замыкании R 3 цепь будет состоять из R 1 параллельно с R 2 , а общее сопротивление будет:

    Контрольные вопросы

    1. Рассчитайте общее сопротивление для каждой из следующих цепей резисторов:
      1. Последовательная цепь: R1 = 40 Ом, R2 = 75 Ом
      2. Параллельная цепь: R1 = 200 Ом, R2 = 200 Ом, R3 = 200 Ом
      3. Последовательная цепь: R1 = 2000 Ом, R2 = 6000 Ом, R3 = 2200 Ом
      4. Параллельная цепь: R1 = 14 Ом, R2 = 32 Ом
      5. Последовательная цепь: R1 = 4700 Ом, R2 = 800 Ом, R3 = 200 Ом
      6. Параллельная цепь: R1 = 60 Ом, R2 = 30 Ом, R3 = 15 Ом
    2. 9 0007 Резисторы R 1 , R 2 и R 3 (50 Ом, 30 Ом и 20 Ом соответственно) подключены последовательно через приложенное напряжение 200 В, чтобы сформировать делитель напряжения.Используя формулу делителя напряжения, рассчитайте напряжения E 1 , E 2 и E 3 .
    3. Полный ток на два параллельно соединенных резистора составляет 3 А. Сопротивление R 1 составляет 10 Ом, а сопротивление R 2 составляет 40 Ом. Используя формулу делителя тока, рассчитайте токи I 1 и I 2 .
    4. Резистор 5 Ом, R 1 , и резистор 20 Ом, R 2 , подключены параллельно друг другу и последовательно с резистором 6 Ом, R 3 .Вычислите полное сопротивление этой последовательно-параллельной цепи.
    5. Вам дали три резистора 100 Ом, которые нужно соединить вместе. Опишите три возможные конфигурации цепей и вычислите их значения общего сопротивления.

    Обзорные вопросы — ответы

    1. (a) 115 Ом, (b) 66,7 Ом, (c) 10 200 Ом, (d) 9,74 Ом, (e) 5700 Ом, (f) 8,57 Ом
    2. E 1 = 100 В, E 2 = 60 В, E 3 = 40 В
    3. I 1 = 2.4 A, I 2 = 0,6 A
    4. 10 Ом
    5. Три резистора по 100 Ом, подключенных последовательно, общее сопротивление 300 Ом. Три резистора по 100 Ом, подключенные параллельно, общее сопротивление 33,3 Ом. Два резистора 100 Ом, подключенные параллельно, а затем последовательно соединенные с резистором 100 Ом, общее сопротивление 150 Ом.

    [PDF] Расчеты электрических цепей — Скачать бесплатно PDF

    1 Расчет электрических цепей Последовательные цепи Во многих цепях имеется более одного преобразователя (т.е. тост…

    Вычисление электрических цепей Последовательные цепи Во многих цепях имеется более одного устройства преобразования (например, тостер, нагреватель, лампы и т. Д.), А в некоторых имеется более одного источника электроэнергии. Если компоненты схемы соединены встык, образуя единую петлю, это последовательная схема.

    Помните, что ток — это скорость, с которой электроны движутся по цепи. Так же, как в нескольких шлангах, соединенных последовательно, чтобы сформировать одну длинную линию, вода может выходить только на дальнем конце с той же скоростью, с которой она поступает (без учета трения).На каждом компоненте есть падение напряжения (и это сила, необходимая для преобразования электрической энергии в какую-либо другую форму). Все падения напряжения в цепи складываются в ЭДС. (Напряжение) источника питания. В приведенной выше схеме E = V1 + V2 (предполагается, что соединительные провода имеют очень маленькое сопротивление, что не является необоснованным предположением.) Амперметры Амперметры — это инструменты, которые измеряют ток и поэтому должны находиться на пути тока, то есть , последовательно с устройством, ток которого измеряется.

    Падение напряжения на каждом компоненте можно рассчитать с помощью закона Ома, если известны ток цепи и значение сопротивления каждого компонента. (Помните, что ЭДС источника питания распространяется на всю цепь, а не только на какой-либо из компонентов.)

    Рабочий пример: • Если показанная схема потребляет 10 ампер от источника питания, рассчитайте падение напряжения на каждом резисторе и объем ЭДС питания Обратите внимание, что поскольку это последовательная схема, ток во всех резисторах одинаков.

    V1 = IR1 = 10 x 10 = 100 В V2 = IR2 = 10 x 20 = 200 В V3 = IR3 = 10 x 5 = 50 В При падении напряжения все складывается с напряжением питания EE

    = V1 + V2 + V3 = 100 + 200 + 50 =

    Если бы в приведенной выше схеме было задано напряжение питания, но не текущий объем, мы могли бы рассчитать его, потому что полное сопротивление последовательной цепи Rtotal — это просто сумма сопротивлений

    Это общее сопротивление в цепи питания, поэтому эту схему можно заменить одним резистором на 35 Ом.

    350v

    Ток питания

    I

    = E / Rt = 350/35 = 10 ампер

    Посмотрите на схему ниже. Несмотря на то, что мы соединили два резистора, вы сможете сразу определить, какое падение напряжения на двух резисторах вместе.

    Падение напряжения конечно 24 вольта.

    Если указаны ЭДС E и отдельные сопротивления, рассчитайте ток цепи и напряжение на каждом резисторе упадет. 1 Сложите сопротивления, чтобы получить общее сопротивление 2 Рассчитайте ток цепи, разделив мощность E на RT 3 Рассчитайте падение напряжения на каждом резисторе по формуле: V1 = I x R1 V2 = I x R2 V3 = I x R3 и т. Д. .

    A Рабочий пример • Рассчитав приведенную ниже схему, I Общее сопротивление 2 Ток. 3 Падение напряжения на каждом резисторе —

    1. Общее сопротивление:

    R всего

    2. Ток (I) определяется с помощью закона Ома: I

    Следовательно, ток цепи I

    = = =

    R1 + R2 + R3 40 + 5 + 80 125 Ом

    =

    E / RT

    =

    250/125

    =

    2 ампера

    (3)

    Падение напряжения на каждом резисторе: падение напряжения на R1 :

    Падение напряжения на R2:

    Падение напряжения на R3:

    V1

    V2

    V3

    =

    I x R1

    =

    2 x 40

    =

    =

    =

    =

    I x R2

    =

    2×5

    =

    10 вольт

    =

    I x R3

    =

    2 x 80

    =

    160 вольт

    падает на резисторах должно составлять напряжение питания.т.е.

    = VD1 + VD2 + VD3 = 80 + 10 + 160 = 250 вольт.

    Параллельная схема Когда каждый из ряда компонентов схемы подключен к одному источнику питания или между одними и теми же двумя точками в цепи, они считаются подключенными параллельно.

    Когда резисторы включены параллельно, на каждом из них наблюдается одинаковое падение напряжения. На рисунке 1 падение напряжения на R1, R2 и R3 будет одинаковым и будет напряжением питания. На рисунке 2 падение напряжения на E2, R3 и R4 будет одинаковым и составит V2

    В отличие от последовательного соединения, ток в параллельных резисторах не такой же (если только они не имеют одинаковое значение сопротивления).

    Вольтметры Вольтметры — это приборы, используемые для измерения напряжений либо ЭДС, либо падений напряжения. Так как на резисторе (или на выводах батареи или генератора) происходит падение напряжения, вольтметр всегда подключается параллельно нагрузке

    Другой Пример работы • Чтобы измерить ток на каждом резисторе, нам необходимо знать падение напряжения на каждом резисторе.

    Какой ток в IMS? Ответ:

    Другой рабочий пример Изучите эту схему:

    Рассчитайте величину тока в R1.I2

    =

    V2 R2

    =

    100 10

    =

    10A

    I3

    =

    V2 R3

    =

    100 5

    000

    =

    100 5

    = 200005

    V2 R4

    =

    100 5

    =

    20A

    I5

    =

    V2 R5

    =

    100 25

    =

    4A I2 + I3 + I4 + I5 10 + 20 + 20 + 4 54A

    Эквивалентное сопротивление Есть еще одна вещь, которую мы можем узнать о параллельной цепи, ее эквивалентное комбинированное сопротивление. параллельна и заставит тот же комбинированный ток потребляться от источника питания.

    Экспериментально можно показать, что два резистора на 6 Ом и 12 Ом, включенные параллельно, могут быть заменены одним резистором на 4 Ом для измерения тока.

    Расчет эквивалентного сопротивления Если мы подключим омметр к цепи, как показано ниже, он будет измерять эквивалентное комбинированное сопротивление резисторов 6 Ом и 12 Ом, включенных параллельно. Как вы можете видеть, он показывает четыре Ом

    120

    Итак, с параллельными цепями вы не добавляете сопротивления.Чтобы подтвердить показания счетчика, мы можем вычислить эквивалентное сопротивление, используя закон Ома.

    Мы уже нашли полный ток, который будет протекать через эквивалентное сопротивление, и знаем напряжение на нем.

    Использование формулы закона Ома: RT

    =

    EI

    =

    24 6

    =

    4 Ом

    Таким образом, мы заключаем, что параллельное сопротивление 6 Ом и 12 Ом будет действовать так, как если бы они были единичным сопротивлением 4 Ом. Важно отметить, что величина эквивалентного сопротивления.(4 Ом) ниже наименьшего параллельного сопротивления (6 Ом). В параллельной цепи полное сопротивление всегда меньше наименьшего сопротивления.

    Метод второй Другой способ вычисления параллельного сопротивления ряда резисторов, включенных параллельно, основан на формуле 1 Эквивалентное сопротивление

    =

    1 + R1

    1 R2

    +

    1 R3

    Два резистора, R1 2 Ом и резистор 3 Ом подключены параллельно к источнику питания 100 В. Какой ток будет потребляться этой комбинацией от источника питания?

    A Рабочий пример • Изучите схему ниже и сравните ее со схемой на странице 17

    Вы можете видеть, что добавление еще одного резистора на 12 Ом параллельно привело к увеличению общего тока до 8A.Определите эквивалентное сопротивление сейчас. 1 RT

    =

    1 R1

    +

    1 R2

    +

    1 R3

    1 RT

    =

    1 6

    +

    1 12

    +

    +

    + 9000 1 RT

    =

    4 12

    RT

    3 Ом

    Или, для расчета по закону Ома: =

    E IT

    =

    24 8

    =

    3 Ом

    RT

    Эквивалент сопротивление было 4 Ом до того, как в параллель был добавлен резистор на 12 Ом.Таким образом, добавление резистора снизило эквивалентное сопротивление. Например, если добавить еще один турникет на футбольном поле, толпа будет быстрее перемещаться по площадке, или, другими словами, весь поток людей будет испытывать меньшее сопротивление движению со стороны добавление дополнительного турникета В таблице ниже указан порядок, в котором вы должны работать при работе с простыми обычными цепями, в которых указаны значения напряжения питания и отдельных сопротивлений. 1 Определите падение напряжения на каждом параллельном сопротивлении.Падение напряжения на каждом сопротивлении одинаковое. 2 Рассчитайте ток в каждом параллельном сопротивлении, используйте формулу закона Ома: I = V R Двумя известными значениями являются падение напряжения на резисторе и сопротивление резистора.

    3 Определите полный ток, протекающий в цепи. Полный ток равен сумме токов в параллельных резисторах.

    4 Рассчитайте эквивалентное сопротивление Используйте формулу закона Омов: I = V R Два известных значения — это падение напряжения на резисторе и 1T — общий ток.

    Чтобы сложить дроби Для сложения или вычитания дробей (что часто приходится делать для решения проблем параллельной цепи) будут полезны следующие рекомендации:

    Параллельные цепи серии

    серия с одной или несколькими параллельными комбинациями

    В приведенной выше цепи ток будет проходить через последовательный резистор, а затем делиться на A и течь через обе ветви параллельной комбинации. Поскольку через последовательный резистор прошел ток, на нем произойдет падение напряжения.Следовательно, напряжение на параллельных резисторах не будет таким же, как напряжение, приложенное к цепи. Это будет э.д.с. напряжение минус падение напряжения на последовательном резисторе. Чтобы вычислить полное сопротивление в цепи, сначала уменьшите параллельную часть цепи до эквивалентного значения сопротивления, а затем рассмотрите всю цепь как последовательную цепь. Это эквивалентное значение сопротивления представляет собой значение сопротивления, которое заменит подключенные параллельно резисторы.

    • Два резистора R1 и R2 на два Ом и четыре Ом подключаются параллельно, а затем последовательно соединяются с резистором R3 на 4 Ом 1

    Рассчитайте общее сопротивление цепи.

    Эквивалентное сопротивление параллельной ветви = RP 1 = RP

    1 + R1

    1 R2

    1 = RP

    1 2

    1 4

    RP

    =

    3 4

    =

    =

    4 3

    =

    +

    1,3 Ом

    Общее сопротивление цепи = RT RT = R3 + RP = 1,3 + 4 = 5,3 Ом 2

    Если бы эта цепь была теперь подключена к источнику питания 100 В, какой бы на ней ток? брать.

    IT = E RT = 100 5,3 = 18.9A

    2

    V1

    V2

    V2

    Каким будет падение напряжения на каждом резисторе.

    =

    IT x эквивалент R R1 и R2

    =

    18,9 x 1,3

    =

    24,6 В

    =

    IT x R3

    =

    18,9 x 4

    V

    =

    E — V1

    =

    100 — 24,6

    =

    75,4 V

    (Небольшая разница в ответах связана с точностью в десятичных разрядах, но не существенна.)

    Резюме В этом модуле рассмотрены последовательные цепи, в которых резисторы соединены встык, и особенности последовательной цепи: (a) Ток имеет одинаковое значение во всей цепи. (b) «Общее сопротивление последовательной цепи — это сумма сопротивлений. (c) Падение напряжения на каждом резисторе может быть рассчитано с использованием закона Ома, т.е.Падение напряжения = ток цепи x сопротивление (d) Падение напряжения на каждом резисторе при сложении равно напряжению питания (E).Затем была рассмотрена параллельная схема, в которой два или более резистора подключены к источнику питания или между одними и теми же двумя точками в цепи. A

    Падение напряжения на всех резисторах, включенных параллельно, одинаковое.

    B

    Общее сопротивление параллельной цепи RT можно рассчитать по формуле: 1 RP

    =

    1 R1

    +

    1 R2

    +

    1 R3

    C Ток в каждой Резистор может быть рассчитан из падения напряжения на резисторе, деленного на его сопротивление (обратите внимание, что падение напряжения на резисторах будет одинаковым для всех резисторов в параллельной ветви).Если параллельная ветвь — единственное, что есть в цепи, падение напряжения на ней будет таким же, как и напряжение питания E. D Полный ток цепи входит в ветвь, а затем разделяется, чтобы пройти через параллельные резисторы пропорционально их значениям. Таким образом, полный ток цепи — это сумма всех токов в параллельном банке.

    Третий тип схемы — это последовательно-параллельная схема, в которой некоторые резисторы соединены последовательно с параллельными рядами резисторов. В последовательной / параллельной цепи следует помнить следующее: A Общее сопротивление цепи — это сумма резисторов, включенных последовательно, плюс эквивалентные сопротивления всех параллельных групп.B Общий ток в цепи можно рассчитать, разделив напряжение питания E на общее сопротивление цепи или вычислив все токи в любом параллельном блоке и сложив их вместе, или разделив падение напряжения на любом последовательном резисторе на его сопротивление. . C Общее напряжение источника питания E можно рассчитать, сложив все падения напряжения в цепи (помня, что на каждом последовательном резисторе есть падение напряжения, но только у каждой параллельной группы есть одно падение напряжения на нем).

    Расчет общего сопротивления

    Упражнения, в которых рассчитывается общего сопротивления электрических цепей.В некоторых упражнениях применяется соотношение между сопротивлением R [& ohm;] и проводимостью G [S] → G = 1 / R. При анализе электрических цепей постоянного тока часто возникает необходимость рассчитать их полное сопротивление, которое видно на клеммах источника питания. Знание значения общего сопротивления необходимо для расчета значения тока, протекающего по цепи. Значение основного тока, протекающего по цепи, можно рассчитать с применением закона Ома → I = U / R.

    Суммарное сопротивление — пример 1

    Расчет полного сопротивления для конкретной электрической цепи.Электрическая схема в примере имеет смешанные соединения между резисторами. Общее сопротивление видно со стороны клемм источника питания, клеммы помечены метками A и B. Электрическая схема содержит три резистора в своей топологии. Некоторые соединения между резисторами специфичны. Будут отмечены узлы с одинаковым электрическим потенциалом. Как известно, между узлами с одинаковым электрическим потенциалом ток не течет.

    Общее сопротивление — пример 1

    Суммарное сопротивление — пример 2

    Расчет общего сопротивления для конкретной электрической цепи.Схема построена на восьми резисторах. Электрическая схема в примере имеет смешанные соединения между резисторами. Общее сопротивление видно с клемм источника питания. При расчете полного сопротивления применяется соотношение между сопротивлением R [& ohm;] и проводимостью G [S] → G = 1 / R. Некоторые соединения между резисторами специфичны. Будут отмечены узлы с одинаковым электрическим потенциалом. Как известно, между узлами с одинаковым электрическим потенциалом ток не течет. Значение электрического потенциала в конкретных узлах обозначено формулой для делителя напряжения .

    Суммарное сопротивление — пример 2

    Суммарное сопротивление — пример 3

    Электрическая схема построена особым образом, потому что резисторы образуют букву Н. Они похожи на мост Н. Обозначение полного сопротивления этой цепи довольно сложно. Можно представить, что его боковые ответвления создают делители напряжения. Если делители напряжения равны, через резистор R3 не будет протекать ток. В этой конкретной ситуации мы можем опустить резистор R3 при вычислении общего сопротивления и рассматривать схему как разрыв вместо резистора R3.Вы можете загрузить моделирование в pspice и расчет в Excel, чтобы изучить поведение схемы.

    Суммарное сопротивление — пример 3

    Суммарное сопротивление — пример 4

    Электрическая схема построена на пяти резисторах. Все резисторы имеют одинаковое значение, равное 1 [кОм]. Будет вычислено полное сопротивление этой электрической цепи. Общее сопротивление видно между определенными клеммами A и B. Резисторы в цепи соединены смешанным образом, это означает, что они соединены последовательно и параллельно.Соотношение между сопротивлением R и проводимостью G будет применяться при расчетах → G = 1 / R.

    Общее сопротивление — пример 4

    Суммарное сопротивление — пример 5

    Электрическая схема построена из семи резисторов. Все резисторы имеют одинаковое значение, равное 1 [& Ом]. Будет вычислено полное сопротивление этой электрической цепи. Общее сопротивление видно между определенными клеммами A и B. Резисторы в цепи соединены смешанным образом, это означает, что они соединены последовательно и параллельно.Соотношение между сопротивлением R и проводимостью G будет применяться при расчетах → G = 1 / R.

    Суммарное сопротивление — пример 5

    Калькулятор параллельного / последовательного резистора

    | Детали с усилением

    Используйте этот калькулятор для определения общего сопротивления сети. Этот калькулятор может давать результаты для последовательного, параллельного и любого их сочетания. Схема создается автоматически по мере добавления резисторов в сеть в качестве наглядного пособия.

    Сложные резистивные схемы часто можно упростить до одного резистора эквивалентного номинала.В процессе упрощения используются два уравнения: резисторы в последовательном уравнении и резисторы в параллельном уравнении.

    Резисторы серии

    Резисторы включены последовательно при соединении в одну линию. Текущий ток является общим для всех резисторов в этой цепи. Это связано с тем, что ток, протекающий через первый резистор, проходит по одному пути через каждый из следующих резисторов в цепи. Общее сопротивление должно равняться сумме номиналов каждого резистора, используемого в цепи.

    $$ R _ {\ text {Equiv}} = R_1 + R_2 + R_3 + \ ldots R_n $$

    Мы можем рассматривать всю эту цепочку резисторов как один резистор со значением ~ R _ {\ text {Equiv}} ~.

    Параллельные резисторы

    Резисторы включены параллельно, если они используют одни и те же два узла. Падение напряжения на каждом резисторе в этой конфигурации обычное. Теперь ток имеет несколько путей и может не быть одинаковым для каждого резистора. Общее сопротивление резисторов, подключенных параллельно, является суммой, обратной величине каждого используемого резистора.

    $$ \ frac {1} {R _ {\ text {Equiv}}} = \ frac {1} {R_1} + \ frac {1} {R_2} + \ frac {1} {R_3} + \ ldots \ frac { 1} {R_n} $$

    Мы можем рассматривать эти параллельные резисторы как один резистор со значением ~ R _ {\ text {Equiv}} ~

    Обратите внимание, что информация, представленная в этой статье, предназначена только для справочных целей. Amplified Parts не делает никаких заявлений, обещаний или гарантий относительно точности, полноты или адекватности содержания этой статьи и прямо отказывается от ответственности за ошибки или упущения со стороны автора.В отношении содержания данной статьи не дается никаких гарантий, подразумеваемых, выраженных или установленных законом, включая, помимо прочего, гарантии ненарушения прав третьих лиц, права собственности, товарной пригодности или пригодности для определенной цели. или его ссылки на другие ресурсы.

    резисторов последовательно | Формула эквивалентного сопротивления

    Введение

    Резисторы являются основными компонентами любой электрической или электронной схемы. Часто резисторы встречаются в большом количестве независимо от размера схемы.Резисторы могут быть подключены последовательно или параллельно или их комбинация. Чтобы уменьшить сложность различных комбинаций резисторов, необходимо соблюдать некоторые правила.

    Считается, что два резистора включены последовательно, если через них протекает одинаковый ток. Последовательные резисторы можно заменить одним резистором. Все резисторы подчиняются основным законам, таким как закон Ома и текущий закон Кирхгофа, независимо от их комбинации и сложности.

    Резисторы серии

    Считается, что набор резисторов включен последовательно, когда они соединены друг с другом в одну линию.Одинаковый ток будет протекать через все резисторы. Считается, что последовательно соединенные резисторы имеют общий ток.

    В цепи последовательных резисторов величина протекающего тока будет одинаковой во всех точках.

    I R1 = I R2 = I R3 = I AB .

    Рассмотрим следующую последовательную резистивную цепь

    Здесь последовательно соединены резисторы R1, R2 и R3 номиналом 1 Ом, 2 Ом и 3 Ом. Одинаковый ток будет протекать через все резисторы, поскольку они соединены последовательно.Общее сопротивление цепи равно сумме отдельных сопротивлений.

    Если R T — полное сопротивление, то

    R T = R1 + R2 + R3

    Теперь эквивалентное сопротивление цепи

    R EQ = R1 + R2 + R3

    R EQ = 1 Ом + 2 Ом + 3 Ом

    R EQ = 6 Ом

    Теперь последовательно соединенные резисторы можно заменить одним резистором R EQ номиналом 6 Ом.

    Формула эквивалентного сопротивления

    В цепи последовательных резисторов общее сопротивление равно сумме отдельных сопротивлений, поскольку через каждый резистор проходит одинаковый ток.

    RR ИТОГО = R 1 + R 2 + R 3

    Например, рассмотрим два последовательно соединенных резистора, как показано ниже

    Сочетание двух последовательно соединенных резисторов 3 Ом эквивалентно одному резистору 6 Ом.Таким образом, приведенная выше схема такая же, как следующая

    .

    Аналогичным образом рассмотрим три резистора, подключенных последовательно, как показано ниже

    Сочетание трех последовательно соединенных резисторов 3 Ом эквивалентно одному резистору 9 Ом. Следовательно, приведенная выше схема такая же, как и следующая

    Это единичное сопротивление называется эквивалентным сопротивлением цепи и используется для замены любого количества последовательно подключенных резисторов.

    Если в последовательной сети n резисторов, то

    R EQ = R 1 + R 2 + R 3 + …….. + R n

    Наблюдение можно сделать из приведенного выше уравнения. Эквивалентное сопротивление резисторов, соединенных последовательно, всегда больше, чем сопротивление самого большого резистора.

    Расчет напряжения

    Для резисторов, включенных последовательно, напряжение на каждом резисторе не соответствует тому же правилу, что и ток. В случае последовательно включенных резисторов общее напряжение на резисторах равно сумме индивидуальных разностей потенциалов на каждом резисторе.

    В приведенной выше схеме разность потенциалов на каждом резисторе может быть рассчитана по закону Ома.В последовательной цепи протекает ток 1 А. Тогда по закону Ома

    Разность потенциалов на резисторе R1 составляет I × R 1 = 1 × 1 = 1 В.

    Разность потенциалов на резисторе R2 составляет I × R 2 = 1 × 2 = 2 В.

    Разность потенциалов на резисторе R3 составляет I × R 3 = 1 × 3 = 3 В.

    Следовательно, полное напряжение V AB = 1 В + 2 В + 3 В = 6 В.

    Рассмотрим последовательное соединение трех резисторов R 1 , R 2 и R 3 с протекающим через них током I.

    Пусть падение потенциала от A до B будет V. Это падение потенциала является суммой индивидуальных падений потенциала на каждом отдельном резисторе. Тогда по закону Ома

    Падение потенциала на R 1 составляет V R1 = I × R 1

    Падение потенциала на R 2 составляет V R2 = I × R 2

    Падение потенциала на R 3 составляет V R3 = I × R 3

    ∴ V = V R1 + V R2 + V R3

    ∴V = I × R 1 + I × R 2 + I × R 3

    Если эквивалентное сопротивление резисторов, включенных последовательно в вышеуказанной цепи, составляет R EQ , то

    В = I × R EQ

    При наличии n резисторов последовательно R1, R2….Rn, тогда полное напряжение на них является суммой индивидуальной разности потенциалов на каждом резисторе.

    В T = V R1 + V R2 +… .. + V Rn

    В последовательной комбинации резисторов из n резисторов, если значение сопротивления каждого резистора отличается от другого, то потенциал на каждом резисторе будет другим.

    Резисторы

    N в последовательной комбинации, каждый с разным сопротивлением, будут иметь N различных разностей потенциалов на них.Этот тип схемы образует делитель напряжения. Схема делителя напряжения является основой конструкции потенциометра.

    В последовательной цепи значения напряжения, тока или сопротивления можно рассчитать по закону Ома. Резисторы можно менять местами в последовательной цепи, не влияя на общую мощность каждого резистора, ток или общее сопротивление цепи.

    Резисторы в серии Примеры

    1. Рассмотрите следующую схему для расчета полного напряжения между A и B.

    Два резистора R1 и R2 включены последовательно.

    R 1 = 2 Ом и R 2 = 3 Ом

    Ток в цепи I = 5 А

    Отдельные падения напряжения можно рассчитать по закону Ома следующим образом:

    Падение напряжения на резисторе R1 составляет В R1 = I × R 1 = 5 × 2 = 10 В

    Падение напряжения на резисторе R2 составляет В R2 = I × R 2 = 5 × 3 = 15 В

    Общее падение напряжения складывается из отдельных падений напряжения.

    В = В R1 + В R2 = 10 + 15 = 25 В

    Другой подход — вычисление эквивалентного сопротивления последовательной комбинации. Отдельные резисторы в последовательной комбинации могут быть заменены одним резистором эквивалентного сопротивления. Эквивалентное сопротивление двух последовательно включенных резисторов R1 и R2 составляет

    Ом.

    R EQ = R 1 + R 2 = 2 + 3 = 5 Ом

    Тогда по закону Ома

    Падение напряжения на A и B равно

    В = I × R EQ = 5 × 5 = 25 В.

    1. Рассмотрим следующую схему, в которой значение индивидуального падения потенциала на каждом резисторе указано вместе с током в последовательной комбинации. Общее сопротивление последовательной комбинации составляет R = 30 Ом. Сила тока в цепи — 1 А.

    R = 30 Ом и I = 1 А

    Ток, протекающий через каждый резистор, одинаков.

    I = I1 = I2 = I3 = I4 = 1 А.

    По закону Ома значение сопротивления можно рассчитать как

    R 1 = V 1 / I 1

    R 1 = 5/1 = 5 Ом

    Аналогично R 2 = V 2 / I 2

    R 2 = 8/1 = 8 Ом

    И R 3 = V 3 / I 3

    R 3 = 7/1 = 7 Ом

    Потенциал по R 4 не указан.Но значение R 4 можно рассчитать исходя из значения общего сопротивления или эквивалентного сопротивления цепи.

    R EQ = R 1 + R 2 + R 3 + R 4

    4 рэнд = рэнд EQ — ( рэнд 1 + р 2 + р 3 )

    Р 4 = 30 — (5 + 8 + 7)

    R 4 = 10 Ом

    Теперь потенциал по 4 рэнд может быть рассчитан как

    В 4 = I 4 × R 4

    ∴V 4 = 1 × 10 = 10V

    Общее напряжение V AB можно рассчитать двумя способами.

    Первый метод основан на использовании индивидуальных разностей потенциалов.

    Общее напряжение равно сумме индивидуальных разностей потенциалов.

    V AB = V 1 + V 2 + V 3 + V 4

    Где V1, V2, V3 и V4 — разности потенциалов на резисторах R1, R2, R3 и R4 соответственно.

    Следовательно, V AB = 5 + 8 + 7 + 10

    В AB = 30 В

    Второй метод расчета общего напряжения заключается в использовании значения эквивалентного сопротивления.

    Полное напряжение равно произведению силы тока на эквивалентное сопротивление. Значения полного тока и эквивалентного сопротивления даны как I = 1 A и R EQ = 30 Ом.

    Следовательно, V AB = I × R EQ

    В AB = 1 × 30

    В AB = 30 В

    Приложения

    Когда два резистора с разным сопротивлением соединены последовательно, напряжение на них разное. Этот метод является основой схем делителей напряжения.

    Если один резистор в цепи делителя напряжения заменить датчиком, то измеряемая величина преобразуется в электрический сигнал, который легко измерить. Часто используемые датчики — это термисторы и светозависимые резисторы. В термисторе сопротивление изменяется в зависимости от температуры. Например, предположим, что термистор имеет сопротивление 10 кОм при температуре 25 0 C. Тот же термистор может иметь сопротивление 100 Ом при температуре 100 0 C.Следовательно, падение потенциала на термисторе будет различным в зависимости от температуры. Это изменение сопротивления в зависимости от температуры можно откалибровать, чтобы найти значение температуры по падению потенциала на термисторе.

    Рис: Цепь датчика освещенности

    Другой датчик, в котором последовательно используются резисторы, — это фоторезистор или светозависимый резистор. В светозависимых резисторах сопротивление изменяется в зависимости от интенсивности падающего на них света.В отсутствие света сопротивление типичного светозависимого резистора достигает 1 МОм. В присутствии света сопротивление светозависимого резистора падает до небольшого значения, обычно порядка нескольких Ом. Это изменение сопротивления в соответствии с интенсивностью света приведет к различным падениям напряжения. Падение напряжения можно откалибровать, чтобы определить наличие света определенной длины волны.

    12. Параллельные цепи переменного тока

    Вспомните закон Ома для чистых сопротивлений:

    `V = IR`

    В случае цепей переменного тока мы представляем сопротивление (эффективное сопротивление) в виде комплексного числа Z .Единицы равны Ом (Ом).

    В этом случае закон Ома принимает вид:

    В = IZ .

    Напомним также, если у нас несколько резисторов ( р 1 , р 2 , R 3 , R 4 ,…) подключено в параллельно , тогда общее сопротивление R T , выдается по:

    `1 / (R_T) = 1 / R_1 + 1 / R_2 + 1 / R_3 +…`

    В случае цепей переменного тока это становится:

    `1 / (Z_T) = 1 / Z_1 + 1 / Z_2 + 1 / Z_3 + …`

    Простой кейс:

    Если у нас есть 2 импеданса Z 1 и Z 2 , соединены параллельно, затем общая сопротивление Z T , равно

    `1 / (Z_T) = 1 / Z_1 + 1 / Z_2`

    Мы можем записать это как:

    `1 / (Z_T) = (Z_2 + Z_1) / (Z_1Z_2)`

    Находя взаимность обеих сторон, мы получаем:

    `Z_T = (Z_1Z_2) / (Z_1 + Z_2)`

    Пример 1

    Найдите суммарный импеданс следующая цепь:

    Ответ

    Назовите импеданс, заданный верхней частью цепь Z 1 и импеданс, определяемый нижняя часть Z 2 .

    Мы видим, что Z 1 = 70 + 60 j Ом и Z 2 = 40-25 j Ом

    Так

    `Z_T = (Z_1Z_2) / (Z_1 + Z_2)`

    `= ((70 + 60j) (40-25j)) / ((70 + 60j) + (40-25j))`

    `= ((70 + 60j) (40-25j)) / (110 + 35j)`

    (сложение комплексных чисел должно производиться в прямоугольной форме.

    Теперь мы конвертируем все в полярную форму, а затем умножаем и делим как следует):

    `= ((70 + 60j) (40-25j)) / (110 + 35j)`

    `= ((92.текст (o) `

    (Преобразуем обратно в прямоугольную форму.)

    `= 37,22-5,93j`

    (При умножении комплексных чисел в полярной форме мы умножаем члены на (числа на переднем плане) и складываем углы. При делении комплексных чисел в полярной форме мы делим члены на и вычитаем углы. См. раздел «Продукты и коэффициенты» для получения дополнительной информации.)

    Таким образом, мы заключаем, что суммарный импеданс равен

    .

    `Z_T = 37-5.9j \ Omega`

    Пример 2

    Учитывая, что Z 1 = 200-40 j Ом и Z 2 = 60 + 130 Дж Ом,


    найти

    а) полное сопротивление

    б) фазовый угол

    в) общий линейный ток

    Ответ

    a) `Z_T = frac {Z_1Z_2} {Z_1 + Z_2}`

    `= frac {(200-40j) (60 + 130j)} {(200-40j) + (60 + 130j)}`

    `= frac {(200-40j) (60 + 130j)} {260 + 90j}`

    `= frac {(204.@ «A» `

    Пример 3

    Резистор 100 Ом, катушка индуктивности 0,0200 Н и Конденсатор 1,20 мкФ подключены параллельно с схема, состоящая из резистора 110 Ом, включенного последовательно с `2.40 \ mu» F «` конденсатор. Питание `150 \» В «,` 60 \ «Гц» есть подключен к цепи.

    Рассчитайте общий потребляемый ток от источника питания и его фазового угла.

    alexxlab

    Добавить комментарий

    Ваш адрес email не будет опубликован. Обязательные поля помечены *